Quel est le lien entre le PIB et la croissance économique ?

Introduction

1La croissance du revenu est-elle le seul aspect du développement et améliore-t-elle le niveau de bien-être de la population ? De fait, le développement économique est généralement associé à la croissance du PIB par habitant. Les organisations internationales, telles que l’Organisation des Nations unies (ONU), l’Organisation de coopération et de développement économiques (OCDE), la Banque mondiale et le Fonds monétaire international (FMI), distinguent les pays développés, les pays intermédiaires et les pays à faible développement, selon que leur PIB par habitant est au-dessus ou au-dessous de certains seuils. Mais bien entendu, le développement ne se réduit pas au PIB ; il s’agit d’un processus multidimensionnel qui implique non seulement une augmentation quantitative du capital, de la production et de la consommation, mais aussi des changements sociaux et politiques qualitatifs qui élargissent les choix offerts aux individus. Le progrès institutionnel, les droits humains, la démocratie, l’égalité des sexes et d’autres capacités (au sens d’Amartya Sen) font partie du développement. Peut-on atteindre ces objectifs qualitatifs en maximisant le PIB ? Et la croissance du revenu ne produit-elle pas une série d’effets secondaires négatifs qui nuisent au bien-être, tels que les externalités environnementales, la destruction des liens sociaux traditionnels, la concentration de la population dans des centres urbains et suburbains, le développement du stress lié au travail, etc. ?

Show

2La croissance est-elle dépassée (« Is growth obsolete? ») ? Le titre provocateur de l’article de William Nordhaus et James Tobin (1973) questionne le bien-fondé de l’objectif de croissance en tant que moteur du bien-être. Bien que les auteurs répondent à la question par la négative, de nombreux chercheurs en économie et en sciences sociales sont arrivés à la conclusion selon laquelle dans les pays développés, la croissance économique en soi a peu d’incidence sur le bien-être et qu’elle ne doit donc pas constituer l’objectif premier de la politique économique (voir Oswald, 1997). Peut-on étendre cet argument aux pays en développement ? Ou bien doit-on en croire Inglehart et al. (2008) selon qui la croissance matérielle, telle que mesurée par le PIB par habitant, améliore le bien-être dans les pays en développement car elle sort les populations de la pauvreté et de la précarité, mais devient inutile dans les sociétés modernes et « post-modernes », où la survie, considérée comme acquise, fait place au développement humain et à l’expression de soi ?

3Cet article examine la relation entre croissance du PIB et bien-être dans les pays en développement à partir des mesures subjectives de bien-être, à savoir la satisfaction déclarée par les répondants lors des enquêtes réalisées auprès d’échantillons représentatifs des populations nationales dans le monde. On peut, à l’aide de ces mesures, tenter de vérifier si le PIB est réellement synonyme de bonheur.

4La relation entre revenu et bonheur est l’une des questions les plus importantes, mais aussi les plus controversées, de la littérature fondée sur le bien-être subjectif. Dans un article célèbre, Easterlin (1974) s’interrogeait ironiquement : en élevant le revenu de tous, accroîtrait-on le bonheur de tous ? (« Will raising the incomes of all raise the happiness of all? »). Depuis deux décennies, la relation entre revenu et bonheur suscite de vifs débats. Cependant, la plupart des études portant sur cette relation reposent sur des données relatives aux pays développés. Le paradoxe d’Easterlin est-il également valide pour les pays en développement ou bien s’agit-il d’un phénomène propre aux pays riches ?

5Cet article présente un panorama des éléments d’information accumulés depuis vingt ans et illustre une partie des constats grâce à une base de données internationale très couramment utilisée (l’enquête World Values Survey, 1981-2008) contenant les appréciations subjectives des individus quant à leur niveau de satisfaction dans la vie et de bonheur. Dans une première partie, nous présentons la relation entre revenu, croissance du revenu et bien-être subjectif. Dans quelle mesure les profils habituellement observés dans les pays développés valent-ils également pour les pays en développement ? Nous envisageons la possibilité d’un effet de seuil dans les rendements de la croissance en termes de bien-être, ceux-ci étant plus élevés dans les pays à faible revenu que dans les pays à revenu élevé. Nous présentons ensuite les explications classiques du paradoxe d’Easterlin et évaluons leur importance dans les pays en développement. Nous distinguons enfin les effets secondaires positifs et négatifs de la croissance ainsi que les limites au bien-être subjectif inhérentes à la nature humaine (effets de comparaison et d’adaptation). Enfin, nous exposons quelques raisons pour lesquelles nous estimons que les estimations fondées sur des données individuelles (coupes transversales instantanées et panel) sont plus fiables que les séries longitudinales agrégées. Nous concluons à la difficulté de rejeter l’existence d’une corrélation positive entre revenu et bien-être.

1 – Sources et données utilisées dans cet article

6Cet article s’appuie essentiellement sur la littérature existante. Cependant, nous avons ajouté nos propres estimations, faisant appel aux cinq vagues de la célèbre enquête World Values Survey (WVS, 1981-2008), qui couvre 105 pays à revenu élevé, à faible revenu et en transition, représentant 90 % de la population mondiale. Les mesures de bonheur ont été puisées en priorité dans l’enquête WVS et l’enquête European Social Survey (ESS) ; c’est le cas de 250 de nos 368 observations. En complément, nous avons également utilisé l’enquête ISSP (101 observations) et le Baromètre latino-américain (Latinobarometro) de 2002. Ces corpus de données sont tous accessibles sur le site : http://worldvaluessurvey.org. Les mesures de bonheur et de satisfaction dans la vie ont été administrées dans un format identique dans toutes ces enquêtes, avec des traductions équivalentes dans tous les pays. La question sur le bonheur était formulée comme suit : « Si vous deviez réfléchir à votre vie en général en ce moment, diriez-vous que vous êtes globalement ? : 1. Pas du tout heureux ; 2 ; Pas très heureux ; 3. Assez heureux ; 4. Très heureux [1]. Dans l’enquête WVS, la question relative à la satisfaction dans la vie était formulée ainsi : « Tout bien considéré, êtes-vous satisfait de votre vie en ce moment ? : 1 (pas du tout satisfait)… 10 (très satisfait) » [2]. Ces enquêtes sont réalisées auprès d’un échantillon représentatif de la population nationale des pays participants, 1 400 personnes en moyenne étant interrogées à chaque cycle. Nous avons calculé la valeur nationale moyenne des réponses à chacune de ces questions (en les traitant comme des variables continues). Nous avons également créé un indice de souffrance (misery index) défini comme le pourcentage de personnes se déclarant très heureuses ou très satisfaites, moins le pourcentage de personnes se déclarant pas du tout heureuses ou pas du tout satisfaites. Comme les résultats étaient similaires, nous ne présentons que les graphiques fondés sur les mesures moyennes.

7Les mesures de PIB par habitant et de croissance annuelle du PIB sont tirées de Heston, Summers et Aten – Penn World Table. Nous avons utilisé d’autres indicateurs quantitatifs diffusés par la Banque mondiale, tels que l’indice de Gini d’inégalité de revenu, le taux de fécondité des femmes, le taux d’alphabétisation des adultes, l’espérance de vie à la naissance (http://data. worldbank.org/). Les indicateurs de gouvernance qualitatifs proviennent de Freedom House et Polity IV (http://www.qog.pol.gu.se/, http://www.freedomhouse.org, http://www.govindicators.org, http://www.systemicpeace.org/polity/polity4.htm). Toutes ces données sont disponibles dans la banque de données : http://www.worldvaluessurvey.org.

2 – Mesures subjectives de bien-être : pourquoi les utiliser et sont-elles fiables ?

8La qualité fondamentale du bien-être subjectif est qu’il est auto-déclaré : ce n’est pas un tiers qui conçoit un ensemble de critères (revenu, santé, éducation, logement, etc.) afin de caractériser le sort des individus, mais les individus eux-mêmes qui sont invités à se prononcer sur leur qualité de vie. Si certains ont douté de l’utilité des mesures subjectives, nous pensons que des raisons très convaincantes incitent à les intégrer à la boîte à outils des économistes.

9Considérons le niveau de bien-être d’un individu comme la somme adéquatement pondérée de tous les aspects de la vie qui lui importent. Au moins deux obstacles significatifs s’opposent à sa mesure objective. Tout d’abord, il faudrait être certain de couvrir tous les aspects de la vie qui importent à l’individu, et il semble a priori difficile d’en dresser une liste définitive. Ensuite, la construction de l’indice de bien-être suppose d’appliquer une pondération appropriée. Or cette entreprise est d’emblée problématique ; dans le contexte des données agrégées utilisées dans l’Indice de développement humain par exemple, quelle est la valeur de l’alphabétisation en termes d’espérance de vie ? De plus, il est très probable que cette pondération varie d’un individu à l’autre. Il est donc très tentant de contourner ces difficultés en demandant implicitement aux individus d’effectuer eux-mêmes ces calculs afin d’évaluer leur vie.

10Les questions sur le bien-être posées dans ce contexte sont souvent très simples : « Globalement, êtes-vous satisfait de votre vie ? » [3] (British Household Panel Survey, BHPS), question à laquelle on répond sur une échelle de sept points, 1 indiquant « Pas satisfait du tout », 4 « ni satisfait ni mécontent » et 7 « entièrement satisfait » [4]. Les individus peuvent être également sondés sur leur bonheur, comme dans la question suivante de l’enquête américaine General Social Survey (GSS) : « Tout bien considéré, comment diriez-vous que vont les choses pour vous en ce moment, diriez-vous que vous êtes très heureux, assez heureux ou pas très heureux ? » [5]. D’autres questions peuvent faire référence à des affects positifs ou négatifs ou à la santé mentale.

11Ces questions sont de plus en plus utilisées dans les enquêtes dans toutes les sciences sociales. Leur succès tient d’abord à leur facilité d’intégration dans un questionnaire, car la majorité d’entre elles sont des questions uni-dimensionnelles (pour une présentation des échelles multi-dimensionnelles voir http://www.deakin.edu.au/research/acqol/instruments/instrument.php). Le deuxième avantage est que la grande majorité des répondants semblent comprendre la question : les taux de non-réponse sont très bas. Enfin, la troisième raison, et à notre avis la plus importante, c’est que les réponses à ces questions semblent rendre compte de la façon dont les individus se comportent.

12On pourrait craindre en effet que les individus ne comprennent différemment la question ou utilisent différemment les échelles de réponses ; une personne ayant choisi 6 sur une échelle de satisfaction allant de 1 à 7 n’est pas nécessairement mieux lotie qu’une personne qui a choisi l’échelon 5. Heureusement, on dispose aujourd’hui d’un corpus de données diversifié suggérant que ces mesures de bien-être subjectif sont bien porteuses d’informations valides.

13La première remarque qui s’impose est que les mesures de bien-être subjectif « se comportent bien », au sens où nombre des corrélations font sens. En effet, les variables indiquant le mariage, le divorce, le chômage, la naissance d’un premier enfant, etc. présentent généralement la corrélation attendue avec le bien-être subjectif des individus [6]. Si les réponses aux questions de bien-être étaient véritablement aléatoires, il n’en serait pas ainsi.

14La question est de savoir si en demandant à A s’il est heureux et à quel point, on obtient des informations sur son niveau réel, inobservable, de bonheur. Une vérification simple, dite « validation par évaluation croisée » (« Cross-Rater Validity »), consiste à demander à B s’il pense que A est heureux. Ce travail a été effectué dans différents cadres (voir Sandvik et al., 1993, et Diener et Lucas, 1999), notamment en interrogeant les amis, la famille ou même l’enquêteur. Une autre solution consiste à recourir à des individus qui ne connaissent pas le sujet : B peut être invité à visionner un enregistrement vidéo de A ou à lire une transcription d’un entretien ouvert avec A. Dans tous les cas, l’évaluation que fait B du bien-être du répondant correspond bien à la réponse donnée par celui-ci.

15Une autre méthode de validation consiste à lier les scores de bien-être à diverses mesures physiologiques et neurologiques. Il a été démontré que les réponses aux questions sur le bien-être sont corrélées aux expressions faciales telles que le sourire ou le froncement de sourcils, ainsi qu’au rythme cardiaque et à la pression artérielle. La littérature médicale a montré que les scores de bien-être étaient corrélés à des troubles digestifs et à des maux de tête, aux maladies coronariennes et aux attaques. La recherche s’est également intéressée aux mesures physiques de l’activité cérébrale et plus particulièrement aux différences d’activité des ondes cérébrales entre les cortex préfrontaux gauche et droit, le premier étant associé à des sentiments positifs et le second à des sentiments négatifs. Ces différences peuvent être mesurées au moyen d’électrodes placées sur le cuir chevelu ou de scanners. Des recherches ont montré (par exemple, Urry et al., 2004) que ces différences d’activité cérébrale sont corrélées avec les déclarations de bien-être des individus. Ces mesures d’asymétrie du cerveau sont associées au cortisol et à la cortico-libérine (CRH), qui régulent la réaction au stress et à la production d’anticorps à la suite d’une vaccination antigrippale (Davidson, 2004). Conformément à l’idée que le bien-être subjectif et l’asymétrie du cerveau mesurent le même concept sous-jacent, les individus qui se déclarent très satisfaits de leur vie avaient moins tendance à s’enrhumer lorsqu’ils étaient exposés au virus et guérissaient plus vite s’ils l’attrapaient (Cohen et al., 2003).

16Les derniers éléments d’information confortant l’idée que les individus « pensent ce qu’ils disent » sont confortés par les données de panel qui suivent des individus sur une longue période : ceux qui se déclarent insatisfaits d’une certaine situation ont davantage tendance à prendre des mesures observables pour la quitter. Ce phénomène est apparent sur le marché du travail, où la satisfaction dans l’emploi dont l’individu fait état à un moment t, est un bon prédicteur de sa démission ultérieure (exemples : Freeman, 1978, Clark et al., 1998, Clark, 2001, et Kristensen et Westergaard-Nielsen, 2006). Un important constat subsidiaire de cette littérature est que la satisfaction dans l’emploi prédit la démission, même lorsqu’on tient compte du salaire et des horaires de travail de l’individu. Cette prédiction du comportement futur semble fonctionner pour les chômeurs comme pour les personnes occupées. Clark (2003) montre ainsi que les scores de stress mental à l’entrée dans le chômage dans les données de l’enquête BHPS prédisent la durée du chômage, ceux dont le bien-être connaît la plus forte baisse restant le moins longtemps chômeurs. Clark et al. (2010) ont reproduit ce constat à partir des scores de satisfaction dans la vie de l’enquête GSOEP (German Socio-Economic Panel). En dehors du marché du travail, on a montré que les scores de bien-être prédisaient la durée de vie (Palmore, 1969, Danner et al., 2001) et, plus récemment, que les mesures de satisfaction prédisaient les divorces futurs (Gardner et Oswald, 2006, Guven et al., 2010).

17Une partie de l’intérêt de la mesure du bien-être subjectif est de fournir des informations sur les arbitrages entre différents aspects de la vie d’un individu. Si une heure de travail supplémentaire par semaine a le même effet sur le bien-être qu’un supplément de 80 euros de salaire mensuel, le salaire de référence (le salaire qui compenserait une heure de travail supplémentaire) est de l’ordre de 18,50 euros de l’heure. La littérature récente donne des exemples de ces arbitrages de bien-être. Par exemple, à partir de données américaines et britanniques, Blanchflower et Oswald (2004, p. 1381) ont conclu que : « pour compenser l’état de chômage des hommes, leur revenu devrait augmenter en moyenne d’environ 60 000 USD par an. Un mariage durable vaut 100 000 USD par an (comparativement au veuvage ou à la séparation). »

18Cette capacité des données subjectives à pondérer les différentes dimensions du développement entre elles (pour calculer les taux marginaux de substitution entre deux dimensions) est particulièrement adaptée à la multidimensionnalité du développement économique. La structure de l’équation de bien-être estimée dans un pays peut être considérée comme une mesure synthétique qui aurait agrégé les différents arguments d’une fonction de bien-être social. Le problème habituel des responsables de la planification sociale (et de l’école du choix social de l’économie normative) est en effet de décider des poids à attacher aux différents arguments de la fonction d’utilité sociale. Les mesures subjectives évitent cet obstacle en mesurant directement le résultat synthétique de la pondération opérée par les individus eux-mêmes, comme le montre par exemple l’article de Di Tella et MacCulloch (2008, p. 31-33), dans lequel les auteurs utilisent l’enquête américaine GSS et l’Eurobaromètre pour estimer les fonctions de bien-être national. Ils proposent les taux marginaux de substitution suivants :

  • Espérance de vie / revenu : « Une personne qui espère vivre une année de plus du fait de la diminution du risque de mortalité est prête à payer 5 052 USD de revenu annuel en contrepartie (6,6 % du PIB par habitant) ».
  • Espérance de vie / chômage : « En termes de taux de chômage, priver un individu d’une année d’espérance de vie a un coût équivalent à une augmentation de 1,1 point de pourcentage du taux de chômage ».
  • Pollution / PIB : « Une augmentation d’un écart-type des émissions de SOx, égale à 23 kg par habitant, entraîne une diminution du bien-être équivalente à une chute de 15 % du niveau de PIB par habitant. »
  • Inflation /chômage : « Une augmentation de 1 % du niveau d’inflation diminue autant le bonheur qu’une augmentation de 0,3 point de pourcentage du taux de chômage ».
  • Criminalité / PIB : « Une augmentation de la criminalité de 242 à 388 agressions pour 100 000 personnes aux États-Unis (soit une augmentation de 60 %) serait équivalente à une chute d’environ 3,5 % du PIB par habitant. »
  • Temps de travail / PIB : « Une augmentation de 1 % du temps de travail devrait être compensée par une augmentation de 2,4 % du PIB par habitant » (pour que le bonheur reste inchangé).
Ces exemples montrent que les mesures de bien-être subjectif peuvent être d’utiles outils d’une politique publique visant à maximiser le bien-être à mesure du développement des pays.

19Avant d’aborder les données sur la croissance et le bien-être subjectif, il nous faut avertir le lecteur de deux approximations abusives dans notre article. Premièrement, nous utilisons indifféremment les termes bonheur, satisfaction dans la vie et bien-être. Deuxièmement, nous traitons ces mesures comme si elles étaient cardinales alors qu’elles sont ordinales. Nous suivons ainsi, comme la majorité des économistes spécialistes de l’étude du bonheur, la voie ouverte par Ferrer-i-Carbonnell et Frijters (2004).

1 – La relation paradoxale entre croissance et bien-être

20L’un des principaux catalyseurs de l’abondante littérature sur le revenu et le bonheur a été l’article novateur d’Easterlin (1974 ; mis à jour en 1995), qui présentait le « paradoxe » de la forte croissance du revenu réel dans les pays occidentaux observée depuis cinquante ans, mais sans augmentation correspondante des niveaux de bonheur déclarés. Ce constat est paradoxal à plus d’un titre. Premièrement, il met à mal le sentiment populaire selon lequel des gains de richesse matérielle et de liberté de choix doivent s’accompagner d’un plus grand bien-être. D’une certaine façon, l’organisation de nos sociétés repose sur ce principe implicite. Deuxièmement, il contredit un vaste ensemble de données empiriques scientifiques basées sur des analyses transversales internationales et sur des données de panels individuelles nationales. Dans cette partie, nous présentons et analysons les données relatives à ces constats contradictoires et posons la question de savoir si le paradoxe d’Easterlin est un phénomène propre aux pays riches ou si les décideurs politiques des pays en développement doivent également le garder à l’esprit. Un résumé du large éventail de sources de données et des résultats est présenté à l’Annexe 2.

1.1 – Les individus les plus riches se déclarent plus heureux

a – Analyses en coupes transversales nationales

21« À ma connaissance, une relation significative a été observée entre bonheur et revenu dans toutes les enquêtes nationales représentatives réalisées jusqu’ici [7] » (Easterlin 2005, p. 67).

22Dans la quasi-totalité des études empiriques réalisées à partir d’enquêtes nationales, les estimations statistiques du bien-être subjectif comprennent, à titre de contrôle ou de variable d’intérêt, le revenu individuel ou celui du ménage (ou plus précisément, le logarithme du revenu). Le revenu logarithmique attire un coefficient positif et statistiquement significatif d’une ampleur considérable. Il apparaît systématiquement parmi les corrélats les plus importants du bonheur déclaré. « Lorsque nous représentons le bonheur moyen en fonction du revenu moyen de groupes d’individus dans un pays donné à un moment t… les individus riches sont en fait bien plus heureux que les pauvres. L’importance de l’écart est véritablement surprenante. Vous ne pourriez imaginer aucun changement qui puisse faire progresser votre vie sur l’échelle de bonheur autant que de passer de la tranche de 5 % la plus basse de l’échelle de revenus à la tranche de 5 % la plus haute [8] » (Frank, 2005, p. 67). Cela vaut pour les pays développés comme pour les pays en développement, même si l’on a parfois observé que le coefficient de corrélation entre revenu et bonheur est plus élevé dans les pays en développement ou en transition que dans les pays développés (voir Clark et al. 2009 pour une revue).

23Layard et al. (2010) rapportent par exemple que dans un même pays, une augmentation d’une unité du revenu logarithmique accroît généralement le bonheur déclaré par les individus de 0,6 unité en moyenne (sur une échelle à dix niveaux). Stevenson et Wolfers (2008, p. 13) ont estimé la corrélation entre bien-être et revenu dans chacun des pays disponibles dans plusieurs corpus de données internationales (l’enquête américaine General Social Survey, le World Values Survey, le Gallup World Poll, etc.). Ils concluent que « Globalement, le coefficient de corrélation moyen entre bien-être et bonheur est de 0,38, la majorité des estimations se situant entre 0,25 et 0,45 et 90 % se situant entre 0,07 et 0,72. L’hétérogénéité représente sans doute en grande partie de simples variations de l’échantillonnage : l’erreur-type moyenne spécifique à un pays est de 0,07, et 90 % des régressions spécifiques à un pays présentent des erreurs-types allant de 0,04 à 0,11 [9] ».

24À titre d’illustration, la figure 1.A représente la relation entre revenu des ménages et bonheur aux États-Unis. La relation est bien représentée par une fonction log-linéaire. Le même constat a été vérifié dans une série d’enquêtes sur la population de pays en développement. La figure 1.B illustre la relation entre bonheur et décile de revenu en Chine en 2007 (à partir de l’enquête World Values Survey) : la même relation positive est évidente. De manière générale, le fait que dans une société donnée, les riches soient plus heureux que les pauvres est un constat empirique bien établi et non contesté de la littérature.

Figure 1.A

Revenu et bonheur dans l’enquête américaine General Social Survey (1972-2006).

Quel est le lien entre le PIB et la croissance économique ?

Revenu et bonheur dans l’enquête américaine General Social Survey (1972-2006).

Extraite de Stevenson et Wolfers, 2008.
Notes : Chaque cercle agrège la satisfaction dans une catégorie de revenu dans un pays, et son diamètre est proportionnel à la population de cette catégorie de revenu dans ce pays. L’axe vertical indique les coefficients obtenus par l’estimation d’un probit ordonné de la satisfaction dans la vie dans chaque pays, en contrôlant pour les effets fixes pays. L’axe horizontal indique le logarithme de la moyenne du revenu réel des ménages dans chaque pays x moins la moyenne du pays. La courbe en pointillés provient d’une régression MCO pondérée par le nombre de personnes interrogées pour chaque catégorie de revenu x dans le pays.

Figure 1.B

Revenu et bonheur dans une coupe instantanée chinoise

Quel est le lien entre le PIB et la croissance économique ?

Revenu et bonheur dans une coupe instantanée chinoise

Nous avons agrégé les trois déciles supérieurs (7, 8, 9) qui étaient très peu représentés dans l’échantillon chinois.

b – Analyses en coupes transversales internationales

25Concernant la relation entre bonheur et revenu ressortant des études internationales, les données empiriques sont encore plus concluantes et consensuelles. Ainsi, Deaton (2008) rapporte une élasticité de 0,84 entre le log du revenu moyen et la satisfaction nationale moyenne dans un large ensemble d’échantillons représentatifs de la population nationale de 129 pays développés et en développement, recueillis par le Gallup World Poll de 2006. Dans le même esprit, Inglehart (1990, chapitre 1) a analysé les données de 24 pays de niveau de développement différencié et a constaté une corrélation de 0,67 entre le PNB par habitant et la satisfaction dans la vie. Dans un article plus récent, Inglehart et al. (2008) ont observé une corrélation de 0,62 sur tous les cycles disponibles de l’enquête World Values Survey. Wolfers et Stevenson (2008, p. 12), utilisant un corpus de données très complet, font état « d’un coefficient de corrélation entre bien-être et PIB international [..] généralement centré autour de 0,4 [10] » [11]. Dans les enquêtes analysées par Inglehart et al. (2008), 52 % des Danois se déclaraient très satisfaits de leur vie (plus de 8 sur une échelle de 10) et 45 % se disaient très heureux. En Arménie, 5 % déclaraient qu’ils étaient très satisfaits et 6 % se disaient très heureux.

26La figure 2.A (extraite d’Inglehart et al. 2008) illustre la relation concave entre le revenu par habitant et le bonheur moyen dans les pays développés, en développement et en transition pour les années 1995-2007. Un graphique similaire a été établi par Deaton (2008) à partir de l’enquête World Values Survey (1996) et du Gallup World Poll (2006) (figure 2.B). Comme l’illustre la figure 2.C, « chaque multiplication par deux du PIB national est associée à une augmentation constante de la satisfaction dans la vie [12] » (Deaton, 2008). La figure 2.D illustre la relation log-linéaire entre le revenu par habitant et la satisfaction moyenne dans la vie dans les pays du monde à la fin des années 2000, à partir des cycles les plus récents de l’enquête World Values Survey.

Figure 2.A

PIB par habitant et bien-être subjectif dans le monde.

Quel est le lien entre le PIB et la croissance économique ?

PIB par habitant et bien-être subjectif dans le monde.

Extraite de Inglehart, Foa, Peterson, Welzel (2008), p. 269
Note : Le bien-être subjectif (BES), par produit intérieur brut (PIB), et les différents types de sociétés. L’indice de bien-être est basé sur la satisfaction dans la vie et le bonheur rapportés en utilisant les moyennes des résultats de toutes les enquêtes disponibles effectuées entre 1995 et 2007 (R2=0,62).

Figure 2.B

PIB par habitant et satisfaction dans la vie.

Quel est le lien entre le PIB et la croissance économique ?

PIB par habitant et satisfaction dans la vie.

Extraite de Deaton (2008), p. 57.
La satisfaction dans la vie dans les enquêtes du Gallup World Poll et du World Values Survey (les données Gallup World Poll figurent en cercles vides, les données World Values Survey en cercles teintés)
Note : Chaque cercle représente un pays, avec le diamètre proportionnel à la population. Le PIB par habitant en 2003 est mesuré en parité de pouvoir d’achat au prix de base en dollars enchaînés de 2000.

Figure 2.C

PIB par habitant et satisfaction dans la vie

Quel est le lien entre le PIB et la croissance économique ?

PIB par habitant et satisfaction dans la vie

Extraite de Deaton (2008), p. 57.
Chaque doublement du PIB est associé à une croissance constante de la satisfaction dans la vie
Note : Chaque cercle représente un pays, avec un diamètre proportionnel à la population. L’échelle de l’axe x est en logarithme. La courbe du milieu indique la satisfaction dans la vie moyenne pour chaque niveau du PIB par tête tandis que les deux autres courbes de l’extérieur indiquent la même chose, mais pour deux groupes d’âge seulement, les âges allant de 15 à 25 – la courbe du haut dans la plupart des cas – et les âges de 60 et plus – qui est le plus souvent la courbe du bas. Le PIB par habitant en 2003 est mesuré en parité de pouvoir d’achat au prix de base en dollars enchaînés de 2000.

Figure 2.D

PIB par habitant dans les années 2000 et satisfaction dans la vie

Quel est le lien entre le PIB et la croissance économique ?

PIB par habitant dans les années 2000 et satisfaction dans la vie

Le PIB et la satisfaction moyenne sont calculés pour la dernière année disponible de l’enquête de chaque pays (de 2001 à 2008).

27De nombreuses autres études sur la « macroéconomie du bonheur » ont établi que de manière générale, les individus se déclarent plus heureux et plus satisfaits de leur vie dans les pays à revenu élevé (voir par exemple, Blanchflower 2008), même si certains types de sociétés semblent plus propices que d’autres au bonheur (Inglehart et al. 2008). La figure 2.A par exemple montre que les pays d’Amérique latine se situent systématiquement au-dessus de la ligne de régression, alors que les pays en transition forment un groupe situé très au-dessous de la ligne de régression type de l’enquête [13].

Développement et inégalité du bien-être subjectif

28En complément de la relation entre revenu moyen et bonheur moyen, nous avons examiné la relation entre les scores moyens de satisfaction dans la vie et leur écart-type (en traitant cette mesure comme une variable continue). Les comparaisons internationales conduisent à une observation frappante : plus le score moyen national de bonheur est élevé, plus l’écart-type interne au pays est faible. Les pays les plus riches présentent à la fois des scores moyens plus élevés et des écarts-types plus faibles de satisfaction dans la vie (figure 3). Ceci indique un bénéfice potentiellement important de la croissance du PIB pour les pays à faible revenu : si les individus sont averses au risque, réduire la variance du bien-être subjectif dans une société donnée est un objectif intéressant de politique publique.

c – Une relation positive sur données de panel individuelles

29Grâce à la disponibilité croissante d’enquêtes de panel auprès de la population de plusieurs pays, une série d’études a pu neutraliser les effets fixes individuels non observés tels que les traits de personnalité. Toutes concluent à une corrélation positive entre la variation du revenu réel et celle du bonheur (voir, par exemple, Winkelmann et Winkelmann, 1998 ; Ravallion et Lokshin, 2002 ; Ferrer-i-Carbonell et Frijters, 2004 ; Senik, 2004, 2008 ; Ferrer-i-Carbonell, 2005 ; Clark et al., 2005). De plus, plusieurs de ces études ont pu s’appuyer sur des variations exogènes du revenu pour établir plus fermement l’effet causal du revenu individuel sur le bonheur (par exemple, Gardner et Oswald, 2007 ; Frijters et al., 2004a, 2004b, 2006, Pischke, 2010). Bien entendu, la pente de la relation revenu-bonheur n’est pas nécessairement identique d’un groupe à l’autre (Clark et al., 2005 ; Frijters et al., 2004a ; Lelkes, 2006). Le coefficient sur les variations intra-individuelles du log du revenu est généralement proche de 0,3 (Layard, Mayraz et Nickell, 2010 ; Senik 2005).

30Les données nationales et internationales montrent donc que les riches sont plus heureux que les pauvres au sein d’un même pays, que les habitants des pays riches sont en moyenne plus heureux que ceux des pays pauvres, et qu’une augmentation du revenu individuel dans le temps est associée à une augmentation positive du bonheur subjectif. À ce stade, les données plaident fortement pour une politique de développement axée sur la croissance du PIB dans les pays à faible revenu.

Figure 3

PIB par tête, bonheur moyen et écart type du bonheur

Quel est le lien entre le PIB et la croissance économique ?

PIB par tête, bonheur moyen et écart type du bonheur

1.2 – Les rendements décroissants de la croissance du revenu

31Cependant, comme l’illustrent les figures 1 et 2, la relation positive entre revenu et bonheur se caractérise par des rendements décroissants. Ce phénomène n’est pas surprenant pour les économistes, habitués à l’idée de la concavité des préférences, c’est-à-dire à l’utilité marginale décroissante et à l’aversion au risque. Concrètement, cela signifie que l’effet d’un gain de revenu de dix mille dollars sur le bien-être subjectif s’affaiblit progressivement à mesure qu’augmente le niveau initial de revenu. Cette observation est cohérente avec la bonne correspondance entre la relation revenu-bonheur et la forme fonctionnelle logarithmique – un phénomène bien connu des spécialistes.

a – Y a-t-il un seuil dans l’utilité de la croissance ?

32

« Dès lors qu’un pays a plus de 15 000 USD annuels par habitant, son niveau de bonheur semble indépendant de son revenu par habitant [14] »

33De nombreux spécialistes croient à l’existence d’un seuil dans l’effet bien-être du revenu. Ils reconnaissent que l’on est plus heureux dans les pays riches que dans les pays pauvres, mais estiment qu’au sein des pays riches il n’y a plus de relation entre PIB par habitant et bonheur. Ce seuil, qui distingue les « sociétés de survie » (« survival societies ») des « sociétés modernes » (« modern societies ») (Inglehart et al. 2009), est généralement supposé dans un intervalle compris entre 10 000 USD et 15 000 USD par an (Di Tella et al. 2007) [15]. Layard (2005, p. 149) écrit ainsi : « Si nous comparons les pays, aucune donnée ne montre que les pays riches sont plus heureux que les pauvres – tant que nous nous limitons aux pays dont le revenu est supérieur à 15 000 USD par an… Il en va différemment au-dessous de 15 000 USD par habitant … [16] » Frey et Stutzer (2002, p. 416) affirment de même que « le revenu apporte le bonheur aux faibles niveaux de développement mais lorsqu’un seuil (voisin de 10 000 USD) est franchi, le niveau de revenu moyen du pays a peu d’effet sur le bien-être subjectif moyen [17] ».

34De manière encore plus explicite, Inglehart (1997, pp. 64-65) conclut que : « la transition d’une société de famine à une société de sécurité apporte une augmentation considérable du bien-être subjectif. Mais nous observons un seuil à partir duquel la croissance économique ne semble plus accroître le bien-être de manière significative. Ce phénomène peut être lié au fait qu’à ce niveau, la plupart des individus ne craignent plus de mourir de faim. On commence à considérer la survie comme acquise […] Aux niveaux faibles de développement économique, des gains économiques même modestes offrent un rendement élevé en termes d’apport calorique, de vêtements, d’abri, de soins médicaux et, à terme, d’espérance de vie.. […]. Mais dès qu’une société atteint un certain seuil de développement… on atteint un point au-delà duquel le supplément de croissance économique n’apporte que des gains modiques d’espérance de vie et de bien-être subjectif. Les situations restent encore très diverses d’un pays à l’autre, mais à partir de ce point, les aspects non économiques de la vie acquièrent une influence croissante sur l’espérance et la qualité de vie [18] »… Les auteurs parviennent à la même conclusion avec des données plus récentes : « Le bonheur et la satisfaction dans la vie augmentent fortement lorsqu’on passe du seuil de subsistance à un niveau modeste de sécurité économique, puis il se stabilise. Dans les sociétés riches, de nouvelles augmentations du revenu n’ont qu’un lien faible avec des niveaux plus élevés de bien-être subjectif [19] » (Inglehart et al., 2008, p. 268).

35L’implication de ces conclusions pour les pays en développement serait que la croissance du PIB doit être considérée comme un objectif temporaire, à ne prendre en compte que jusqu’à un certain point.

b – Cependant, le coefficient de corrélation entre bonheur et log du PIB par habitant ne tend pas vers zéro

36Malgré ces affirmations, les données internationales en faveur d’un tel seuil de subsistance sont loin de faire consensus. Réunissant plusieurs corpus de données d’enquêtes internationales couvrant environ 90 % de la population mondiale dont de nombreux pays en développement (à partir des enquêtes World Values Survey et Gallup World Poll), Stevenson et Wolfers (2008, p. 11-12) ont tenté de vérifier l’hypothèse d’un seuil de 15 000 USD par habitant par an (en dollars constants de 2000). Ils ont estimé la corrélation entre bonheur et PIB par habitant et observé que : « la corrélation entre bien-être et PIB est à peu près deux fois plus forte pour les pays pauvres que pour les pays riches. C’est-à-dire […] qu’une hausse de 100 USD du revenu est associée à une augmentation du bien-être à peu près deux fois plus forte pour les pays pauvres que pour les pays riches [20] ». Cependant, l’utilité marginale de la croissance du PIB reste positive dans les pays développés. « Les estimations ponctuelles sont en moyenne environ trois fois plus élevées pour les pays dont le revenu est supérieur à 15 000 USD que pour ceux dont le revenu est inférieur à 15 000 USD. […] A priori, les résultats de Gallup suggèrent que l’effet sur le bien-être mesuré d’une augmentation de 1 % du PIB par habitant serait à peu près trois fois plus élevé dans les pays riches que dans les pays pauvres. Bien entendu, une augmentation de 1 % du PIB par habitant des États-Unis correspond à un montant environ dix fois supérieur à celui d’une augmentation de 1 % du PIB par habitant de la Jamaïque [21] ».

37Cela rejoint l’analyse que fait Deaton des mêmes données du Gallup World Poll (figure 2.B) : « la relation entre le logarithme du revenu par habitant et la satisfaction dans la vie est pratiquement linéaire. Le coefficient est de 0,838, avec une faible erreur-type. Un terme quadratique dans le logarithme du revenu a un coefficient positif : ce qui confirme que la pente est plus élevée dans les pays riches ! […] Si l’on retient un seuil de 12 000 USD de revenu par habitant pour différencier les pays riches et les pays pauvres, la pente dans les pays à revenu élevé est plus forte / […] Si des indications pointent vers un écart, il est faible et probablement dans le sens d’une pente plus forte dans les pays à revenu élevé [22] ».

38Deaton (2008) conclut que « la pente est plus forte au sein des pays les plus pauvres, où les gains de revenu sont associés aux augmentations les plus fortes de satisfaction dans la vie, mais elle reste positive et substantielle même dans les pays riches ; il n’est pas vrai qu’il existe un seuil critique de PIB par habitant au-dessus duquel le revenu n’a plus d’effet sur la satisfaction dans la vie [23] ». Autrement dit, l’utilité marginale de la croissance du PIB décroît bien à mesure de l’augmentation du niveau du PIB par habitant, mais le rendement de la croissance ne tend pas vers zéro [24].

39En résumé, tous les travaux sur le bonheur fondés sur des coupes transversales de pays constatent unanimement que la relation entre revenu par habitant et bonheur est concave, c’est-à-dire qu’elle offre des rendements décroissants. Mais il n’y a pas de consensus sur l’existence d’un seuil de subsistance au-delà duquel l’utilité marginale du revenu tomberait à zéro.

1.3 – « L’utilité marginale du revenu n’est pas décroissante mais nulle » (Easterlin)

40La critique la plus forte de la politique de croissance repose sur les données empiriques relatives à l’évolution à long terme du PIB et du bonheur au sein des pays. Les graphiques élaborés par Easterlin et ses co-auteurs (1974, 1995, 2005, 2007, 2009, 2010) illustrent l’horizontalité de la courbe d’évolution du bonheur sur longue période. L’une des courbes horizontales les plus célèbres et les plus spectaculaires, empruntée à Easterlin et Angelescu (2007), est représentée à la figure 4.A. Bien que le PIB par habitant des États-Unis ait été multiplié par deux en trente ans (1972-2002), le bonheur moyen des Américains est resté constant. Le bonheur moyen est calculé en utilisant les coupes transversales répétées de l’enquête américaine General Social Survey. Plusieurs études ont fait apparaître le même type de profil, avec des séries longues de données relatives à différents pays développés (voir Diener et Oishi, 2000). L’affirmation étayée par ces graphiques est radicale. Pour reprendre les termes de Richard Easterlin : « L’utilité marginale du revenu n’est pas décroissante mais nulle [25] » (Easterlin et Angelescu, 2007, p. 8).

41L’absence de corrélation sur longue période entre croissance et bonheur pourrait s’expliquer par l’utilité marginale décroissante découverte en coupe transversale. Pourtant, Easterlin rejette cette interprétation avec force : « On concilie généralement la constance habituelle du bien-être subjectif face à l’augmentation du PIB par habitant avec les données transversales instantanées en arguant que les séries longitudinales pour les nations développées correspondent à la fourchette supérieure de revenu des coupes transversales, où le bonheur change peu voire pas du tout à mesure qu’augmente le revenu réel. » Mais « l’évolution du revenu dans le temps à l’intérieur de la fourchette de revenu utilisée dans les coupes instantanées ne génère pas la variation de bonheur impliquée par ces dernières [26] » (Easterlin et Angelescu, 2007, p. 24). Ainsi : « En 1972, la cohorte de 1941-1950 avait un revenu par habitant moyen d’environ 12 000 USD (exprimé en prix constants de 1994). En 2000, le revenu moyen de la cohorte avait plus que doublé pour atteindre près de 27 000 USD. Selon la relation instantanée, cette augmentation aurait dû accroître le bonheur moyen de 2,17 à 2,27. En réalité, le bonheur de la cohorte n’a pas changé [27] ».

Figure 4.A

Le paradoxe américain. Bonheur et PIB réel par habitant, États-Unis, 1972-2002

Quel est le lien entre le PIB et la croissance économique ?

Le paradoxe américain. Bonheur et PIB réel par habitant, États-Unis, 1972-2002

Extraite de Easterlin et Angelescu (2007).

42Dans plusieurs articles écrits avec différents co-auteurs (Easterlin, 2005a, Easterlin et Sawangfa 2005), Easterlin a souligné avec force qu’il était impossible de transposer les données transversales à une relation dynamique. L’évolution dynamique sur longue période du bonheur moyen auto-déclaré dans un pays n’est pas correctement prédite par la relation transversale instantanée entre revenu par habitant et bonheur. Dès lors, « connaître la relation instantanée du bien-être subjectif avec le PIB par habitant n’ajoute rien à la capacité à prédire l’évolution du bien-être subjectif dans un pays au cours du temps [28] » (Easterlin et Sawangfa, 2009, p 179). Ce phénomène est illustré par la figure 4.B, extraite d’Easterlin (2005a, p. 16), qui met en contraste l’évolution effective (plate) du bonheur au Japon et l’évolution prédite (log linéaire) dans le temps.

Figure 4.B

Trompeuses coupes transversales. Bonheur réel et prédit au Japon. 1958-1987.

Quel est le lien entre le PIB et la croissance économique ?

Trompeuses coupes transversales. Bonheur réel et prédit au Japon. 1958-1987.

Extraite de Easterlin (2005).

43Il s’ensuit que la relation concave positive entre le PIB par habitant et le bien-être subjectif, que l’on observe en coupe transversale, ne peut être utilisée pour prédire l’évolution temporelle du bien-être subjectif dans les pays en développement. Cette nouvelle théorie du « no bridge » pointe l’erreur qui consiste à transposer les relations transversales à des séries longitudinales. Les pays en développement ne devraient donc pas s’attendre à ce que leur croissance leur permette d’atteindre le niveau de bien-être plus élevé qui caractérise les pays développés.

1.4 – La corrélation dynamique est-elle suffisamment faible pour être écartée ?

44Malgré les graphiques spectaculaires proposées par Easterlin, son rejet de toute corrélation dynamique entre croissance et bonheur suscite encore une très vive controverse. L’un des points d’achoppement porte en particulier sur la question de savoir si le coefficient de corrélation entre le bien-être subjectif avec le PIB par habitant est statistiquement significatif et d’une ampleur conséquente. Il est faible mais est-il « suffisamment faible pour être négligé » ? (Hagerty et Veenhoven, 2000, p. 4).

45Ainsi, l’absence de corrélation entre la croissance et le bonheur dans deux pays ayant connu un rapide développement, le Japon (après la Seconde Guerre mondiale) et la Chine (après 1980), est particulièrement décevante. Cependant, Stevenson et Wolfers (2008) ont relevé des discontinuités dans la formulation de la question sur le bonheur et dans l’échantillonnage des coupes transversales japonaises utilisées par Easterlin. S’agissant de la Chine, les données sont peu nombreuses (trois points dans le temps) et Hagerty et Veenhoven (2000) ont souligné que l’échantillon chinois n’était pas représentatif de la population, car il était initialement biaisé vers des groupes démographiques plutôt urbains.

46D’autres études sur longue période ont conclu à une relation dynamique positive entre le PIB par habitant et le bien-être. Exploitant l’enquête World Values Survey, Hagerty et Veenhoven (2006) ont observé une augmentation statistiquement significative du bonheur dans quatre pays à revenu élevé sur huit, et dans trois pays à faible revenu sur quatre. Inglehart et al. (2008) ont également exploité les cycles les plus récents de l’enquête World Values Survey, couvrant la période de 1981 à 2005. Ils ont observé que sur la période entière, le bonheur avait augmenté dans 45 des 52 pays pour lesquels on dispose de séries temporelles substantielles. Utilisant les données de 21 pays développés et en transition, Kenny (2005) a effectué des régressions de l’évolution du bonheur sur la variation du PIB pour chaque pays et constaté que 88 % des coefficients de corrélation étaient positifs. Le coefficient de régression général était positif et significatif, à 5 %.

47Inglehart et al. (2008) présentent une série de graphiques illustrant l’évolution temporelle du bonheur moyen dans différents pays à partir des quatre premiers cycles de l’enquête World Values Survey. Comme ils le soulignent : « dans de nombreux cas, les résultats infirment l’hypothèse selon laquelle en dépit de la croissance économique et d’autres changements, les populations de sociétés données ne sont pas devenues plus heureuses. Ils montrent que les séries américaines et britanniques affichent une tendance à la baisse du bonheur de 1946 à 1980, mais une tendance haussière par la suite [29] » [cela a été confirmé par Easterlin]. « De manière générale, parmi les pays pour lesquels nous disposons de données sur longue période, le niveau de bonheur augmente dans 19 pays sur 26. Dans plusieurs de ces pays – Corée du Sud, Inde, Irlande, Mexique et Porto Rico – on observe des tendances haussières fortes. Les autres pays affichant une tendance haussière sont l’Afrique du Sud, l’Argentine, le Canada, la Chine, le Danemark, l’Espagne, la Finlande, la France, l’Italie, le Japon, le Luxembourg, les Pays-Bas, la Pologne et la Suède. Trois pays (États-Unis, Suisse et Norvège) affichent des tendances stables de la première enquête à la plus récente. Quatre pays seulement (Allemagne de l’Ouest, Autriche, Belgique et Royaume-Uni) affichent des tendances baissières [30] » (Annexe à Inglehart et al., 2008). Les figures 5.A à 5.E extraites de leur article illustrent la pente positive de la courbe de bonheur en Afrique du Sud, en Inde, au Mexique et à Porto Rico et la tendance baissière en Chine.

48Les figures 5.A à 5.E sont extraites de Inglehart et al. (2008, annexe statistique).

Figure 5.A

Tendance du bonheur en Inde

Quel est le lien entre le PIB et la croissance économique ?

Tendance du bonheur en Inde

Figure 5.B

Tendance du bonheur au Mexique

Quel est le lien entre le PIB et la croissance économique ?

Tendance du bonheur au Mexique

Figure 5.C

Tendance du bonheur à Porto Rico

Quel est le lien entre le PIB et la croissance économique ?

Tendance du bonheur à Porto Rico

Figure 5.D

Tendance du bonheur en Afrique du Sud

Quel est le lien entre le PIB et la croissance économique ?

Tendance du bonheur en Afrique du Sud

Figure 5.E

Tendance du bonheur en Chine

Quel est le lien entre le PIB et la croissance économique ?

Tendance du bonheur en Chine

49Plusieurs études ont ainsi fait apparaître une corrélation positive et statistiquement significative entre la croissance et le bien-être dans le temps dans des séries longitudinales de données nationales, notamment Hagerty et Veenhoven (2003), Stevenson et Wolfers (2008), Inglehart et al. (2008). Easterlin (2005) a critiqué nombre de ces résultats pour le choix des pays, la confusion entre dynamique sur longue période et cycle économique, et l’absence de contrôles dans certaines des estimations. Easterlin, avec plusieurs co-auteurs, a documenté et développé sa conjecture initiale. Des auteurs tels qu’Ed Diener, Rafael Di Tella, Bruno Frey, Robert MacCulloch, Andrew Oswald et Alois Stutzer ont fourni d’autres données empiriques allant dans le sens d’Easterlin.

Remarque sur la puissance statistique

50La controverse sur la relation entre revenu et bonheur sur longue période porte sur l’ampleur du coefficient de corrélation et sa signification statistique. Certains auteurs ont souligné la moindre puissance statistique des séries longues sur le bien-être, par rapport aux données transversales instantanées, en raison d’un écart-type moins élevé. La variation à expliquer étant moins élevée, il est difficile d’obtenir des corrélations statistiquement significatives.

51Hagerty et Veenhoven (2000, p. 5) remarquent par exemple que : « l’écart-type du PIB par habitant dans la coupe transversale de Diener et Oishi était d’environ 8 000 USD alors que dans la série temporelle de Hagerty (pour les mêmes pays) il n’était que d’environ ¼ de cette valeur (dollars de 2000) […] dans un même pays en vingt-cinq ans [31] ». La capacité statistique à détecter l’effet est donc plus faible dans les études de séries longitudinales. De même, Kenny (2005), utilisant les données relatives à 21 pays développés et en transition, a constaté un écart-type de bonheur dans le temps à l’intérieur des pays de 0,28 en moyenne, à comparer avec un écart-type des scores moyens internationaux de 0,65 (p. 212). Layard et al. (2010, p. 161), qui utilisent la série longitudinale de l’Eurobaromètre pour 20 pays d’Europe occidentale, font également état d’un écart-type moyen des scores nationaux de bonheur de 0,2 à comparer à une moyenne de 0,5-0,6 pour les coupes transversales individuelles.

52Nous avons calculé l’écart-type du bonheur et de la satisfaction dans la vie au sein des coupes transversales de l’enquête World Values Surveys de 1981 à 2007. L’écart-type moyen à l’intérieur d’une coupe transversale instantanée est de 0,67 pour le bonheur (quatre modalités) et de 2,14 pour la satisfaction dans la vie (échelle de 10 degrés). Mais l’écart-type du bonheur national moyen tous pays confondus est de 0,28 pour le bonheur et 1,04 pour la satisfaction dans la vie. Enfin, l’écart-type du bonheur national dans le temps fluctue autour de 0,1 pour le bonheur et de 0,13 à 0,41 pour la satisfaction dans la vie. Autrement dit, la variabilité des mesures de bien-être subjectif est beaucoup plus basse dans les séries longitudinales que dans les coupes transversales au sein d’un pays et en comparaisons internationales. De ce fait, la différence entre les coefficients de corrélation obtenus par coupes transversales et les coefficients obtenus par séries longitudinales est difficile à interpréter.

53En résumé, la relation à long terme entre la croissance du PIB et le bien-être subjectif demeure controversée. Comme l’ont remarqué Stevenson et Wolfers (2008), on ne peut pas écarter l’hypothèse nulle d’un coefficient de corrélation égal à zéro, mais il ne s’ensuit pas qu’on peut rejeter l’hypothèse nulle d’un coefficient supérieur à zéro. La nature de la relation sur longue période entre le PIB et le bien-être est loin d’être fermement établie.

1.5 – Bien-être subjectif et cycle économique

54L’une des raisons pour lesquelles il est difficile d’admettre l’absence de corrélation entre le revenu et le bien-être est que cette absence est en forte contradiction avec l’indéniable effet du cycle conjoncturel.

55Tout d’abord, les spécialistes s’accordent largement à penser qu’une récession rend les gens malheureux. Di Tella et al. (2003) ont montré que les mouvements macroéconomiques, en particulier le chômage, l’inflation et la volatilité de la production, exerçaient une forte influence sur le bonheur des nations. L’impact négatif de la volatilité sur le bien-être subjectif a également été établi par Wolfers (2003). Une excellente illustration de la corrélation entre cycle économique et bonheur est donnée par la figure 6.A, extraite de Stevenson et Wol-fers (2008), qui montre la spectaculaire dynamique parallèle de l’écart conjoncturel de production et du bonheur moyen aux États-Unis de 1972 à 2008. Cela ne signifie pas que l’influence du cycle économique peut être assimilée à celle de la croissance sur longue période : il est facile d’imaginer que le bonheur et le cycle économique fluctuent autour d’une tendance stable à long terme.

Figure 6.A

Bonheur et cycle économique.

Quel est le lien entre le PIB et la croissance économique ?

Bonheur et cycle économique.

Extraite de Stevenson et Wolfers (2008)
Notes : « l’écart de production » est la différence entre le PIB réel par tête et sa ten-dance, estimée en utilisant un filtre de Hodrick-Prescott sur des données annuelles du logarithme du PIB réel par tête avec un paramètre de lissage de 6,25. Les données sur le bonheur sont agrégées en un indice de bonheur en effectuant une régression en probit ordonné du bonheur sur les effets fixes années. Voir la figure 8 pour plus de détails sur la question. Voir le texte pour plus de détails sur l’échantillon.

56Le processus de transition des pays d’Europe centrale et orientale du socialisme au capitalisme constitue un épisode particulier, dont on considère généralement qu’il illustre la corrélation entre les fluctuations du revenu et le bien-être plutôt qu’entre la croissance à long terme et le bien-être. Toutes les études reconnaissent unanimement la corrélation statistiquement significative entre la dynamique du PIB et celle du bien-être subjectif. Les figures 6.B à 6.D, extraites de Guriev et Zhuravskaya (2008) et d’Easterlin (2009), illustrent ce type d’évolution concomitante du revenu et du bonheur dans plusieurs pays en transition. Des données similaires peuvent être observées dans Sanfey et Teksoz (2008).

Figure 6.B

Bonheur et transition en Russie.

Quel est le lien entre le PIB et la croissance économique ?

Bonheur et transition en Russie.

Extraite de Guriev et Zhuravskaya
Échelle de Gauche : satisfaction dans la vie pour un individu moyen à partir des régressions de panel avec effets fixes individuels et autres contrôles habituels (intervalle de confiance : 95 %). Échelle de droite : PIB réel par habitant en dollars USD de 2000 corrigé des PPP.

Figure 6.C

Bonheur et transition dans plusieurs pays.

Quel est le lien entre le PIB et la croissance économique ?

Bonheur et transition dans plusieurs pays.

Extraite de Easterlin (2009) La satisfaction dans la vie, 1990, 1995, et 1999, et l’indice du PIB réel annuel 1986-1999 a.
a Ex-Allemagne de l’Est, 1989-2005.

Figure 6.D

Satisfaction dans la vie, 1990-2005, et indice du PIB réel dans trois pays en transition

Quel est le lien entre le PIB et la croissance économique ?

Satisfaction dans la vie, 1990-2005, et indice du PIB réel dans trois pays en transition

57Cependant, Easterlin (2009) qualifie ces évolutions de court terme et met en garde de ne pas « confondre une association bonheur-revenu positive de court terme, due à des fluctuations des conditions macroéconomiques, avec la relation de long terme. Nous formulons l’hypothèse que cette disparité entre l’association de long terme et l’association de court terme est due au phénomène socio-psychologique de l’“aversion à la perte” [32] ».

58Si intéressante soit l’interprétation en termes d’aversion à la perte, l’assimilation de la transition à un phénomène de court terme nous paraît surprenante. D’une certaine façon, la transition est le meilleur exemple imaginable d’un changement de régime. Il s’agit d’une transformation structurelle profonde et irréversible, et non un phénomène de courte durée. Elle présente les caractéristiques essentielles du développement, y compris la période de décollage et les changements qualitatifs et institutionnels profonds. La question reste donc posée de savoir s’il convient de traiter la transition comme une évolution de court terme ou de long terme. Ce n’est qu’avec le passage du temps que l’on pourra observer si l’augmentation du bien-être subjectif se poursuit avec la croissance du PIB, stagne à un certain point ou retrouve son niveau initial (1990). Pour l’instant, la leçon de la transition est que le développement et la croissance du revenu par habitant s’accompagnent d’une augmentation du bien-être subjectif.

2 – Explications liées à la croissance elle-même : canaux et effets secondaires négatifs

59La « platitude » des courbes de bonheur n’indique pas seulement que la croissance économique ne porte pas la promesse d’un accroissement du bien-être dans le temps. Plus généralement, elle indique que quelle que soit la tendance agrégée que connaît une société, elle n’accroîtra pas le bonheur moyen de la population sur le long terme. Si cela est vrai, les perspectives sont très sombres pour les pays en développement, voués à conserver leur niveau actuel de bien-être. Le message est également très décourageant pour les politiques publiques en général : s’il est impossible d’accroître le bonheur sur longue période, non seulement il faut abandonner la croissance en tant qu’objectif, mais cela est vrai de toute mesure de politique publique.

60Avant d’adopter hâtivement ces conclusions radicales, les deux parties qui suivent présentent les explications possibles de l’horizontalité de la courbe de bonheur. Une première série d’explications a trait à la nature même de la croissance, c’est-à-dire aux canaux reliant croissance et bien-être, et au fait que la croissance s’accompagne d’externalités négatives (pollution, inégalités) qui compensent ses bénéfices subjectifs. Une autre série d’explications porte sur les processus sociaux et psychologiques tels que les comparaisons et l’adaptation, qui diminuent les bénéfices subjectifs de la croissance.

2.1 – Qualité de vie : canaux de la croissance du PIB au bien-être

61Les estimations statistiques du bien-être comprennent le plus souvent des effets fixes d’année ou de pays, ainsi que d’autres contrôles introduits pour neutraliser les changements intervenus dans la composition démographique de la population (en termes d’âge, de profession, de santé, du nombre d’enfants, etc.). Certaines estimations neutralisent également les variables politiques telles que la démocratie, l’égalité des sexes, la confiance, etc. Cependant, en termes de stratégie empirique pour estimer une relation entre deux valeurs, il y a toujours un arbitrage entre la neutralisation des variables qui canalisent le phénomène étudié, au risque de ne pas saisir son effet, et l’absence de neutralisation de ces variables, au risque d’obtenir une mesure biaisée. Dans le cas de la croissance et du bien-être, la neutralisation des effets secondaires positifs risque de masquer l’impact de la croissance elle-même. De fait, si la croissance est censée apporter un plus grand bien-être, ce n’est pas seulement par la voie d’un plus grand pouvoir d’achat (revenu), c’est-à-dire l’augmentation des quantités de biens consommés, mais aussi par la transformation sociale tout entière qui accompagne le processus de croissance.

62L’augmentation du revenu par habitant s’accompagne toujours d’un gain de productivité du travail, ce qui donne davantage de choix aux individus concernés quant à l’utilisation du temps. Comme le soutient Sen (2001), c’est parce qu’elle accroît la liberté de choix (en élargissant leurs capacités) qu’on attend de la croissance qu’elle augmente le bien-être des populations. De même, on sait que la croissance du PIB s’accompagne d’une transition démographique dans les pays peu développés. Cela procure certainement « un accroissement révolutionnaire de liberté pour les femmes [33] », pour reprendre les termes de Titmuss (1966, cité par Easterlin et Angelescu 2007, p. 9), ainsi qu’un gain d’éducation et de ressources disponibles pour le développement personnel des enfants. La croissance s’accompagne aussi d’une plus longue espérance de vie, d’une diminution de la mortalité infanto-juvénile et de l’insuffisance pondérale des enfants (voir par exemple Becker, Philipson et Soares, 2005 ou Easterlin et Angelescu, 2007). Enfin, on sait bien que démocratie et développement vont de pair, même si le sens de la causalité n’est pas aussi clair qu’on le pensait au xviiie siècle (par exemple Montesquieu, Steuart et Hume). Lipset (1959, p. 80) affirme ainsi que : « l’industrialisation, l’urbanisation, le niveau élevé d’éducation et une augmentation régulière de la richesse globale de la société [sont] les conditions élémentaires de la démocratie [34]. » Sans inférer aucune causalité, on peut observer l’association statistique entre la croissance du PIB et les progrès en matière de liberté politique et de droits humains. Dès lors, en termes de stratégie empirique, afin de saisir l’effet global de la croissance du PIB sur le bien-être subjectif, il ne faut introduire aucune variable qui neutralise l’effet des canaux véhiculant ces effets. Il est regrettable que nombre des études consacrées à la relation entre croissance du PIB et bonheur intègrent ces contrôles.

63Les paragraphes qui suivent examinent les données disponibles sur la corrélation entre la croissance du PIB et les indicateurs de qualité de vie. Il s’agit de mesures des dimensions quantitatives et qualitatives hors revenu, qui constituent les canaux reliant croissance du revenu et bien-être.

a – Corrélation entre croissance du PIB et indicateurs de qualité de vie en coupe transversale

64Easterlin et Angelescu (2007) ont illustré la forte corrélation positive en coupe transversale entre une série d’indicateurs de qualité de vie et le PIB par habitant, dans un ensemble de pays de niveau de développement diversifié. Des courbes clairement haussières lient le bien-être à des facteurs quantifiables, mesurés sur une échelle continue, tels que : alimentation, logement, vêtements et chaussures, apport calorique, apport protidique, fruits et légumes, radios, automobiles, postes de télévision, abonnés au téléphone mobile, utilisateurs d’Internet, population urbaine, espérance de vie à la naissance, taux brut de scolarisation, taux de fécondité totale. De nombreux autres auteurs ont documenté ces évolutions et leur relation avec le bien-être subjectif ; voir par exemple Inglehart et Welzel (2005), Inglehart et al. (2008), Layard et al., 2010, di Tella et MacCulloch (2008), Becker et al. (2005).

65Certains auteurs ont insisté sur la relation entre les procédures, la gouvernance et les institutions, les droits démocratiques et humains, la tolérance aux autres, l’égalité des sexes d’une part, et le bien-être subjectif d’autre part (par exemple Barro 1997, Frey et Stutzer 2000, Inglehart et Welzel 2005, Schyns 1998, Inglehart et al. 2008).

b – Corrélation entre croissance du PIB et indicateurs de qualité de vie dans les séries longitudinales

66La figure 7 illustre le décollage spectaculaire de l’espérance de vie en Angleterre et au Pays de Galles au xixe siècle. Plus généralement, Easterlin et Angelescu (2007) font un exposé minutieux des progrès enregistrés dans les différentes dimensions de la qualité de la vie dans un large échantillon de pays développés et émergents. Ils illustrent les différentes dimensions de l’évolution de la qualité de vie pendant la « croissance économique moderne », celle-ci étant définie comme « une augmentation rapide et nourrie de la production réelle par habitant et les changements concomitants des techniques de production, des besoins en facteurs de production, et de l’allocation des ressources d’une nation [35] », où l’augmentation « rapide et nourrie » est définie comme étant au moins égale à 1,5 % par an (Easterlin et Angelescu, 2007, p. 2). Ils documentent les tournants décisifs de la croissance du PIB et d’autres indicateurs de la qualité de vie. Bien que les deux variables évoluent dans le même sens, ils insistent sur le fait que les dates de leur décollage respectif ne coïncident pas systématiquement. Les indicateurs qualitatifs ont parfois du retard et parfois de l’avance sur le décollage économique. « Si les indicateurs sociaux et politiques de la qualité de vie sont, à présent, positivement associés au PIB par habitant, c’est souvent parce que les premiers pays à mettre en œuvre les nouvelles technologies de production à la base de la croissance économique moderne sont aussi les premiers à avoir introduit, souvent par le biais de politiques publiques, de nouvelles avancées des connaissances dans le domaine social et politique [36] » (Easterlin et Angelescu, 2007, p. 21). Quant à savoir si les co-mouvements entre la croissance et les indicateurs de qualité de vie sont de nature causale, la question reste controversée et difficile à établir (voir aussi Easterly (1999)). Il est indéniable cependant qu’il n’y a pas de progrès de la qualité de vie sans croissance du PIB.

Figure 7

Le décollage de l’espérance de vie

Quel est le lien entre le PIB et la croissance économique ?

Le décollage de l’espérance de vie

Extraite de Easterlin et Angelescu (2007)

67Dans leur article provocateur intitulé « Is growth obsolete? », William Nordhaus et James Tobin (1973) ont plaidé pour un autre indicateur intégrant les loisirs, les tâches ménagères et les coûts de l’urbanisation et ont construit une « mesure de bien-être économique » (« Measure of Economic Welfare »). Cependant, il s’est avéré que cet indice croissait comme le PIB sur la période étudiée, quoique plus lentement. À notre connaissance, c’est une observation universelle. Pritchett et Summers (1996) observent par exemple la forte corrélation entre le fait d’être riche et en bonne santé » (« wealthier is healthier ») sur longue période. Utilisant des données longitudinales de multiples pays, ils observent que « l’élasticité au revenu sur longue période de la mortalité juvénile et infantile dans les pays en développement se situe entre 0,2 et 0,4 ». Cela implique que « plus d’un demi-million de décès d’enfants dans le monde en développement pour la seule année 1990 peuvent être attribués aux médiocres performances économiques des années 1980 [37] ».

68En résumé, la croissance économique s’accompagne d’une série d’améliorations quantitatives et qualitatives non monétaires. Celles-ci constituent les canaux par lesquels la croissance agit sur le bien-être ; ces derniers ne devraient pas être neutralisés dans les estimations économétriques de la relation.

2.2 – Effets secondaires négatifs de la croissance

69Il est possible que les graphiques PIB-bonheur doivent leur forme plate à l’influence négative de certains effets secondaires de la croissance comme la pollution, les inégalités de revenu, le stress au travail, etc. L’influence de ces « variables omises » pourrait masquer l’influence positive de la croissance du PIB sur le bien-être subjectif dans les estimations économétriques (voir Di Tella et MacCulloch, 2008).

70Les effets secondaires négatifs de la croissance les plus largement illustrés sont les inégalités, la criminalité, la corruption, l’allongement de la durée de travail, la pollution et les autres dégradations environnementales (telles que mesurées par les émissions de SOx par exemple) (Di Tella et MacCulloch, 2003, 2008). Kenny (2005) pointe également le coût social de la transformation économique et le glissement du niveau local au niveau mondial des préoccupations de revenu relatif. On soupçonne également l’impact de la concentration urbaine et du développement des banlieues. Easterlin et Angelescu (2007) soulignent l’effet des émissions de dioxyde de carbone et des apports lipidiques (obésité et hypertension). Clark et Fischer (2009) proposent un utile résumé des corrélats macroéconomiques de la satisfaction dans la vie dans les pays de l’OCDE.

71Parmi les suspects habituels, les inégalités de revenu tiennent une place à part. Premièrement, la relation entre inégalités de revenu et bien-être subjectif a fait l’objet de nombreuses études, dont la plupart ont conclu à une corrélation négative (voir Senik, 2009 pour une revue de la littérature, ou Clark et al. 2008, ou encore Alesina et la Ferrara, 2008 pour une étude de la demande de redistribution du revenu). Les inégalités de revenu peuvent réduire le bien-être si les individus ont une pure aversion à leur égard (bien qu’elles puissent accroître le bien-être de ceux qui les considèrent comme reflétant l’étendue des possibles : voir Alesina et al., 2004). Cependant, elles peuvent aussi exercer un effet mécaniquement dépresseur sur la mesure du bien-être subjectif moyen du fait de la forme fonctionnelle logarithmique de la relation entre revenu et bien-être subjectif (voir Stevenson et Wolfers, 2008). Cet effet technique ne semble néanmoins pas suffisant pour expliquer l’horizontalité de la courbe. Comme l’illustrent les différents graphiques de la figure 8 (extraite de Layard et al., 2010, p. 142), aux Etat-Unis, les inégalités de revenu se sont fortement creusées depuis 1970, le bonheur moyen est resté stable, le revenu du quintile supérieur a augmenté, mais pas son bonheur. Même pour le quintile supérieur de revenu, la courbe de bonheur est restée « plate » aux États-Unis.

72Il faut souligner que les externalités négatives de la croissance suivent souvent une forme de U inversé, c’est-à-dire qu’elles augmentent aux premiers stades du développement et diminuent par la suite. Les inégalités de revenu, la pollution, les longues heures de travail, les mauvaises conditions de travail, etc. ont initialement gagné en importance puis ont commencé à s’atténuer à un certain stade dans les pays à revenu élevé. Ce phénomène ne résulte pas de forces purement mécaniques, mais des politiques publiques – une remarque importante pour les pays en développement.

73Ces facteurs négatifs doivent-ils être pris en compte dans l’évaluation de la croissance du PIB en termes de bien-être ? La question reste ouverte. S’ils constituent des compagnons inévitables de la croissance, la réponse est oui : il faut les compter comme des éléments négatifs dans l’évaluation de la croissance en termes de bien-être. Cependant, s’il est possible de les atténuer ou de les supprimer par le biais des politiques publiques, on peut alors les séparer de l’effet bien-être de la croissance (en les introduisant comme contrôles dans les estimations économétriques).

Figure 8

Croissance du PIB, inégalités et bonheur.

Quel est le lien entre le PIB et la croissance économique ?

Croissance du PIB, inégalités et bonheur.

Extraite de Layard, Mayraz et Nickell (2010, p. 142)
Figure 4 : le bonheur moyen et la moyenne du logarithme du PIB aux Etats-Unis par quintile de revenu (du plus élevé au plus faible). Source : General Sociale Survey, 1972-2006. Le revenu est ajusté pour le nombre d’adultes dans le ménage.

3 – Explications liées à la fonction de bonheur elle-même (les êtres humains sont des animaux sociaux)

3.1 – Comparaisons de revenus

74Une explication simple à l’absence de relation de long terme entre le revenu et le bien-être tient à la structure même de la fonction individuelle de bien-être. L’idée générale est que le revenu n’est pas apprécié hors de tout contexte, mais qu’il s’agit d’une catégorie sociale, évaluée par rapport à une valeur de référence, à l’occasion de comparaisons. Le revenu de référence peut être envisagé comme le revenu moyen d’un groupe ou le juste niveau de revenu. Abstraction faite des autres déterminants, on peut alors écrire la relation entre utilité et revenu ainsi :

75

Quel est le lien entre le PIB et la croissance économique ?

76Le bien-être de l’individu i au moment t augmente avec son revenu yit, mais diminue avec le niveau du revenu de comparaison, yit*. Le revenu de comparaison agit ici comme un déflateur au regard du revenu individuel, au sens où plus il est élevé, moins ce dernier paraît satisfaisant. Une part importante de la littérature empirique explorant cette relation a explicitement paramétré la fonction de bien-être comme une fonction de yit, et de yit/yit*. Si l’effet du revenu sur le bien-être est pour l’essentiel absolu, de sorte que sans les externalités mentionnées plus haut, une hausse du PIB accroîtra le bien-être individuel, alors le second terme ne jouera qu’un rôle mineur. A l’inverse, si les comparaisons de revenus sont très importantes, de sorte que la plus grande partie de l’effet de revenu passe par le revenu relatif, alors c’est le second terme qui sera prépondérant. Dans ce dernier cas, en réponse à la question d’Easterlin, augmenter les revenus de tous n’accroîtra pas le bonheur de tous.

77Au cours des quinze dernières années, de nombreux travaux empiriques ont tenté de différencier ces deux scénarios. Diverses méthodes empiriques dans diverses disciplines ont été mobilisées pour répondre à la question de l’importance des comparaisons de revenus dans la détermination du bien-être. Tous ces travaux ont dû définir a priori précisément à qui ou à quoi les individus se comparaient : le conjoint, les personnes présentant les mêmes caractéristiques que l’individu, celles qui habitent dans la même région, qui exercent le même emploi, ou même une mesure du revenu anticipé de l’individu. Les paragraphes qui suivent décrivent les principaux constats dans les pays développés et en développement.

a – Comparaisons de revenu dans les pays développés

78Une approche directe de la question des comparaisons de revenus consiste à estimer des équations de bien-être dans lesquelles apparaissent à la fois le revenu de l’individu et le niveau du revenu de comparaison : ce sont les contreparties empiriques de l’équation (1) ci-dessus. Cette littérature fait appel à différents corpus de données (en termes de pays et d’années), à différentes mesures de bien-être (la satisfaction dans l’emploi et la satisfaction dans la vie étant les plus fréquentes) et à diverses mesures du revenu de comparaison, yit*. La conclusion type est que le bien-être est positivement corrélé avec le revenu personnel, mais négativement corrélé avec le revenu des autres.

79Clark et Oswald (1996) utilisent l’enquête BHPS pour calculer le revenu de référence professionnel à l’aide d’une équation de salaire, et montrent que celui-ci est négativement corrélé avec la satisfaction individuelle dans l’emploi. Le revenu individuel attire un coefficient positif, et la somme des deux coefficients de revenu est nulle : augmenter le salaire de tous n’accroîtrait la satisfaction de personne. Ferrer-i-Carbonell (2005) et McBride (2001) ont réalisé des travaux dans le même esprit utilisant respectivement des données allemandes et américaines. Vendrik et Woltjer (2006) étendent l’analyse des données de l’enquête GSOEP allemande à cet égard, en considérant les relations asymétriques aux gains et aux pertes (relativement au groupe de référence).

80Une autre mesure de yit* concerne le niveau local : que gagnent mes voisins ? Blanchflower et Oswald (2004) et Luttmer (2005) calculent le revenu moyen régional à l’aide de données américaines et montrent qu’il est négativement corrélé avec le « bien-être » des répondants : un individu qui gagne 40 000 USD par an est plus heureux dans une région pauvre que dans une région riche. À l’échelon local, Clark et al. (2009) constatent, à l’aide des données de panels danoises, que la satisfaction individuelle présente une forte corrélation positive avec le rang de l’individu dans la répartition des revenus locaux. D’autres travaux ont étudié les comparaisons au revenu des collègues (Brown et al., 2006), des époux (Clark, 1996) ou des parents (McBride, 2001).

81Estimer des équations de bien-être n’est qu’une façon parmi d’autres d’aborder la question des comparaisons de revenus. L’une des premières méthodes employées (la première contribution publiée étant celle de Van Praag, 1971) est celle de l’Ecole de Leyden : les individus attribuent des niveaux de revenus à des qualifications verbales de leur niveau de vie (telles que « excellent », « bon », « suffisant » et « mauvais »). À partir de ces valeurs déclarées sont estimées des relations entre bien-être et revenu, incluant parfois un revenu de référence. Les résultats (par exemple, Van de Stadt et al., 1985) montrent que pour un revenu individuel donné, plus le revenu du groupe de référence est élevé, plus les individus déclarent avoir besoin d’argent pour atteindre un niveau donné de bien-être.

82L’économie expérimentale offre d’autres données sur les comparaisons. Dans Zizzo et Oswald (2001), les participants à une expérience utilisent leurs propres gains afin de détruire les gains d’autres participants. Une autre méthode consiste à demander aux individus de choisir entre des revenus hypothétiques, comme dans Alpizar et al. (2005), Johannsson-Stenman et al. (2002) et Solnick et Hemenway (1998). Le choix type se présente comme suit :

83A : Votre revenu annuel actuel s’établit à 50 000 USD ; les autres gagnent 25 000 USD.

84B : Votre revenu annuel actuel s’élève à 100 000 USD ; les autres gagnent 200 000 USD.

85L’élément crucial ici est que l’une des options offre un niveau de revenu absolu plus élevé tandis que l’autre est plus avantageuse en terme relatif. Les travaux expérimentaux montrent l’existence de fortes préoccupations relatives, car les individus préfèrent le plus souvent A à B. La même méthode peut être appliquée pour comparer l’importance des comparaisons selon les domaines. À titre d’exemple, les préoccupations relatives dans Alpizar et al. se révèlent plus fortes pour les voitures et le logement que pour les vacances et l’assurance.

86Une récente expérience randomisée réalisée par Card et al. (2010) atteste de l’effet des comparaisons de revenu sur les employés de l’Université de Californie, pour ceux qui ont eu accès, sur Internet, à des informations concernant le salaire de leurs collègues.

87Enfin, des travaux neurologiques récents utilisent des techniques d’IRM pour mesurer l’activité cérébrale de paires d’individus engagés dans des tâches identiques. Dans Fließbach et al. (2007), la récompense financière de chaque individu pour une bonne réponse était annoncée aux deux sujets et cette récompense était variable. Dans certaines conditions, une bonne réponse donnée par un participant lui faisait gagner 60 points ; dans d’autres, une bonne réponse lui faisait gagner 60 points et la bonne réponse de l’autre participant lui faisait gagner 30, 60 ou 120 points. La rémunération relative de l’individu pour une bonne réponse était donc variable, alors que la rémunération absolue restait fixe. L’analyse de l’oxygénation sanguine a montré que l’activité cérébrale dans le striatum ventral augmentait avec un revenu relatif. Takahashi et al. (2009) évoquent des travaux apparentés dans ce domaine.

b – Comparaisons de revenu dans les pays à bas revenu

88La majorité des travaux sur les comparaisons de revenus et le bien-être individuel concernent les pays de l’OCDE. Cependant, le corpus croissant de données subjectives, conjugué à l’intérêt grandissant des chercheurs pour ces questions, a produit un nombre modeste mais croissant d’informations sur les corrélats du bien-être des individus dans les pays pauvres. Les préoccupations de statut sont-elles moins importantes dans les pays pauvres : les comparaisons sont-elles un luxe ?

89En ce qui concerne l’estimation directe du bien-être individuel, Graham et Felton (2006) ont reproduit le constat d’un effet négatif du revenu régional sur le bien-être individuel dans 18 pays d’Amérique latine. Kuegler (2009) analyse des données collectées en 2005 auprès de 400 Vénézuéliens et montre que ceux qui se déclarent mieux lotis que leur fratrie se disent plus satisfaits de leur vie. Cette observation est cohérente avec les effets de revenu relatif dans un pays relativement pauvre. La puissance de cette corrélation dépend des caractéristiques propres de l’individu ; elle est plus forte pour les individus dont le revenu est supérieur au revenu médian et pour ceux qui exercent des professions de niveau plus élevé. Stark et Taylor (1991) présentent des éléments attestant indirectement du rôle des comparaisons de revenus en examinant la décision de migrer. À partir de données mexicaines, ils montrent que la pauvreté relative prédit bien la décision de migrer du Mexique vers les États-Unis.

90Castilla (2010) examine lui aussi des données mexicaines, notamment des informations sur la pauvreté subjective (le revenu de la personne interrogée suffit-il à ses besoins ?) et la satisfaction à l’égard du revenu. On mesure les dépenses de la personne interrogée et on recueille l’évaluation qu’elle fait de son revenu par rapport aux personnes avec lesquelles elle vit, par rapport à ce qu’elle aspirerait à posséder à ce stade de sa vie et par rapport au revenu qu’elle gagnait trois ans auparavant (ces trois aspects étant mesurés sur une échelle à sept degrés). Les résultats empiriques montrent que le bien-être augmente avec les dépenses, mais chute avec le revenu relatif et l’écart entre le revenu et les aspirations. En ce qui concerne le revenu passé, les résultats ne sont significatifs que dans l’équation de satisfaction dans la vie et lorsque l’individu rapporte être dans une moins bonne situation que trois ans auparavant (ce qui est conforme à l’hypothèse d’aversion à la perte).

91Rojas et Jiménez (2007) recourent eux aussi à des données mexicaines pour montrer que l’évaluation subjective de pauvreté des personnes interrogées est en partie déterminée par les écarts entre leur revenu d’une part et le revenu de comparaison d’autre part. Le revenu de comparaison est mesuré directement en interrogeant les individus sur l’écart de leur revenu « par rapport à ceux auxquels ils se comparent habituellement ». Guillen-Royo (2010) analyse de petits échantillons de données provenant de sept communautés au Pérou et montre que la satisfaction à l’égard de plusieurs domaines de la vie est positivement corrélée avec les dépenses de la personne interrogée mais négativement corrélée avec les dépenses moyennes de la communauté. Enfin, Rojas (2010) utilise des données de 20 pays d’Amérique latine figurant dans l’enquête Gallup de 2007. Deux mesures du bien-être individuel, la satisfaction dans la vie et la satisfaction à l’égard du niveau de vie, sont liées à la fois au revenu individuel et au revenu moyen du groupe de référence (défini par âge, par sexe et par pays). Les résultats empiriques montrent que le bien-être croît avec le log du revenu individuel mais chute avec le log du revenu de comparaison. Dans le cas de la satisfaction à l’égard du niveau de vie, les coefficients des deux variables sont égaux et opposés, ce qui indique qu’une augmentation du revenu de tous n’accroîtrait le bien-être de personne en Amérique du Sud.

92En Asie, une série de travaux a été récemment menée sur les déterminants du bien-être en Chine, dont certains ont fait appel à la notion de revenu de référence. Appleton et Song (2008) concluent que la satisfaction dans la vie déclarée par les Chinois urbains est affectée par des considérations de statut relatif, tandis que Smyth et Qian (2008), analysant les données de 31 villes chinoises en septembre 2002, constatent que le logaritme du revenu mensuel moyen dans la ville où habite la personne interrogée est négativement corrélé avec le bonheur, après neutralisation des effets du revenu individuel. Gao et Smyth (2010) recourent à deux corpus de données pour présenter certains éléments attestant que la satisfaction dans l’emploi est négativement corrélée avec le revenu du groupe de référence, ce dernier étant défini comme le revenu moyen dans l’entreprise qui emploie le répondant, ou le revenu prédit de « personnes comme moi » (à l’instar de Clark et Oswald, 1996).

93Des travaux récents de Cojocaru (2010) font appel à une coupe transversale de 2007 de l’enquête LSMS au Tadjikistan. Ils constatent un effet majoritairement non significatif du revenu régional sur la satisfaction des individus dans la vie, mais suggèrent que cela pourrait indiquer que le groupe de référence utilisé n’est pas adapté. Lorsque l’on utilise une variable qualitative mesurant le jugement porté par un individu sur le bien-être de son ménage relativement à celui de ses voisins, des effets conséquents sont constatés dans le sens attendu : ceux qui situent leur ménage à un niveau relativement bas comparativement à leurs voisins se déclarent moins satisfaits de leur vie, en contrôlant pour les dépenses propres du ménage.

94Fafchamps et Shilpi (2008) analysent une mesure directe de l’utilité relative à l’aide de données népalaises : les réponses à une question sur la suffisance du niveau de consommation du ménage. L’adéquation subjective de la consommation augmente avec le revenu individuel (mais elle diminue avec la distance par rapport au marché le plus proche). La consommation du ménage est jugée d’autant moins satisfaisante que celle du groupe de référence augmente. Ici, la consommation du groupe de référence est définie de façon géographique comme la consommation moyenne ou médiane d’autres ménages vivant dans le même village que la personne interrogée.

95Carlsson et al. (2009) examinent les préférences hypothétiques par rapport à différentes situations de revenu absolu et relatif (comme le font Alpizaret al., 2005) en Inde. Ils observent que la moitié environ de l’effet du revenu sur le bien-être provient d’une préoccupation de statut ou de revenu relatif. La taille de l’effet est à peu près identique à celle des pays riches. En outre, les répondants issus de basses castes et ayant des revenus faibles apparaissent comme les plus sensibles au revenu relatif.

96John Knight a signé une série d’articles utilisant les données chinoises de l’enquête nationale auprès des ménages, CHIP de 2002. Cette enquête comprend non seulement des questions sur le bien-être subjectif, mais également des questions directes sur ce que les personnes interrogées considèrent comme leur groupe de référence. Knight et al. (2009) recourent à une coupe transversale de 9 200 ménages chinois ; ils montrent d’abord que les comparaisons en Chine sont locales : 70 % des individus considèrent leur village comme leur groupe de référence. En outre, ceux qui déclarent que leur revenu est très supérieur à celui du village connaissent des scores de bonheur plus élevés. Knight et Gunatilaka (2010a et 2010b) soulignent eux aussi l’importance du revenu relatif plutôt que du revenu absolu et le rôle des groupes de référence à l’aide de données chinoises. Mishra et al. (2010) montrent que déclarer un revenu inférieur à celui d’un groupe de référence (également auto-déclaré) est associé à un moindre bien-être pour la minorité coréenne en Chine.

97Les travaux sur le bien-être réalisés à partir de données chinoises ont donc produit un faisceau d’indications suggérant la présence de comparaisons de revenu dans les pays en développement. Ils rejoignent les résultats de Brown et al. (2010) relatifs à la consommation ostentatoire dans les pays en développement. Ces derniers utilisent les données d’un panel de ménages chinois et montrent que les dépenses réalisées à l’occasion de funérailles et en cadeaux concordent avec un comportement de recherche de statut. Enfin, Fließbach et ses co-auteurs ont prolongé leurs travaux de 2007 en réalisant les mêmes neuro-expériences sur le revenu relatif en Chine (bien que les résultats ne soient pas encore rédigés).

98Pour ce qui concerne l’Afrique, Kingdon et Knight (2007) envisagent le rôle du revenu relatif en Afrique du Sud. Les auteurs observent des éléments confirmant des effets de revenu relatif au sein des groupes raciaux (plus les autres gagnent, moins on est satisfait de sa vie), mais des effets positifs de revenu dans un voisinage [38].

99Bookwalter et Dalenberg (2010) analysent les données du SALDRU sud-africain du début des années 1990. Ils n’observent pas d’effet significatif du revenu local (niveau de groupe) pour les Blancs mais un effet positif significatif du revenu moyen du groupe pour les non-Blancs. Comme dans les travaux de Cojocaru (2010), le fait de se sentir moins bien loti que ses parents exerce un effet absolu plus important sur la satisfaction que le fait de se sentir mieux loti.

100Ravallion et Lokshin (2010) font appel à des données à grande échelle issues d’une enquête réalisée en 2004 auprès des ménages au Malawi, qui comprend des mesures de satisfaction dans la vie et de dépenses de consommation. Caractéristique inhabituelle, les données comprennent aussi des mesures subjectives du bien-être économique établies à partir des réponses des personnes interrogées à la question « Imaginez une échelle à six marches, où les gens les plus pauvres se situent sur la marche la plus basse, tandis que les plus riches se trouvent sur la marche la plus haute, la sixième. Sur quelle marche êtes-vous aujourd’hui ? » ainsi que leur évaluation du bien-être économique de leurs voisins et amis. Ravallion et Lokshin modélisent la satisfaction des individus dans la vie comme une fonction de la consommation individuelle et de celle du voisinage, et comme une fonction du bien-être individuel et de celui des voisins. Au vu des résultats, ils concluent que les comparaisons ne sont pas importantes pour la majorité des Malawiens ; pourtant il apparaît que la consommation d’autrui diminue la satisfaction dans la vie des individus dans l’échantillon urbain. Certains éléments suggèrent également un effet négatif du bien-être économique des amis sur ceux qui déclarent un niveau relativement élevé de bien-être économique.

101À plus petite échelle, Kenny (2005) utilise les données d’une enquête réalisée auprès de 566 ménages tanzaniens dans laquelle les répondants déclarent le montant de revenu nécessaire pour être heureux. À l’instar des résultats de Van Praag sur données européennes, le revenu régional moyen apparaît comme un déterminant crucial de ce que les individus considèrent comme un niveau de revenu satisfaisant.

102Akay et Martinsson (2008) utilisent une approche « cell-mean » similaire à celle de Ferrer-i-Carbonell (2005) appliquée aux données d’une enquête réalisée en 2004-2005 auprès des ménages dans le nord de l’Éthiopie. Ils ne constatent aucun effet significatif du revenu du groupe de référence sur la satisfaction dans la vie. Un article apparenté (Akay et al., 2009) s’intéresse lui aussi à l’Éthiopie mais en considérant cette fois les préférences hypothétiques pour des scénarios de revenu absolu et relatif. Il s’avère que les choix de la plupart des paysans éthiopiens pratiquant l’agriculture de subsistance reposent sur le seul revenu absolu. Cependant, dans leur échantillon, un nombre non négligeable d’individus – parmi les plus pauvres dans le monde – semblent être affectés par leur position relative.

103Corazzini et al. (2010) recourent à la même approche pour comparer l’importance du revenu relatif dans huit pays différents. S’ils affirment que de manière générale, les individus des pays riches sont plus sensibles au revenu relatif, il est frappant de constater que le Kenya est l’un des pays les plus sensibles aux comparaisons de revenu.

c – Pauvreté absolue et pauvreté relative

104L’un des intérêts des comparaisons de revenus, notamment dans le contexte des pays les moins riches, est qu’elles rejoignent le concept de pauvreté. La distinction entre pauvreté en tant que manque absolu et manque relatif remonte au moins à Adam Smith : au milieu du xviiie siècle, les Écossais dépourvus de chaussures n’étaient pas considérés comme très pauvres, alors qu’en Angleterre, seuls les individus vivant dans le dénuement le plus total n’en portaient pas. Le stigma associé au manque de chaussures était donc plus important en Angleterre qu’en Écosse du fait de la norme sociale qui lui était associée. L’impact d’une carence donnée sur le bien-être individuel peut dépendre du degré de stigmatisation associé à cette dernière au sein de la société, celui-ci étant lui-même probablement lié à la fréquence de la carence en question.

105Pour revenir au revenu et à l’équation (1) ci-dessus, la distinction essentielle est alors de savoir si la pauvreté est définie par le fait que le revenu d’un individu est inférieur à un certain niveau critique ou si le revenu d’autrui joue un rôle. Les mesures absolues de pauvreté reposent sur le coût de l’apport calorique minimum ou d’un panier de consommation minimal (États-Unis). La Banque mondiale fixe le seuil de pauvreté à 1 USD par jour. Les mesures relatives de la pauvreté tiennent compte du contexte, tel le seuil de pauvreté relative couramment utilisé, de 60 % du revenu médian. Les données attestant des préoccupations de statut dans les pays à faible revenu semblent constituer un argument en faveur de mesures de la pauvreté relative.

106Une autre question importante à laquelle nous n’avons pas de réponse à ce jour est de savoir si les préoccupations sont moins importantes, c’est-à-dire si elles ont un effet plus faible sur le bien-être, dans les pays à faible revenu que dans les pays à revenu élevé. On pourrait en effet envisager la perception du revenu d’autrui comme une sorte de bien de luxe, auquel on ne prête attention que lorsque sa survie est considérée comme acquise. Nous avons examiné les études attestant de l’existence de préoccupations relatives dans les pays en développement. Quant à savoir si leur importance y est moindre que dans les pays développés, c’est une question ouverte dont l’étude nécessiterait des données spécifiques – peut-être des données expérimentales. Analysant les données de la troisième vague de l’ESS, Clark et Senik (2010) se sont attachés aux réponses à la question « Vous paraît-il important de comparer votre revenu avec celui des autres ? » [39] dans les pays européens. Ils ont constaté que cette importance était plus grande dans les pays pauvres que dans les pays riches et qu’au sein d’un même pays, cette comparaison était souvent jugée plus importante par les individus pauvres. On se compare le plus souvent « vers le haut » et les individus souffrent davantage de ce type de comparaisons. Cette observation est conforme aux constats généraux de la littérature (voir par exemple Ferrer-i-Carbonnell 2004, ou Card et al. 2010). Si ce constat pouvait être étendu aux pays pauvres, il écarterait l’idée que les comparaisons de revenus sont un phénomène propre aux pays riches.

107Sachant l’importance des comparaisons de revenu à l’échelle locale pour les ressortissants des pays à faible revenu, il y aurait lieu d’envisager également la possibilité que les comparaisons de revenu relatif puissent se faire aussi à l’échelle mondiale, notamment du fait du développement des technologies de l’information et de la communication. Si ces dernières permettent aux habitants des pays à faible revenu de prendre conscience du mode de vie et des possibilités de consommation des ressortissants des pays riches, il est probable que cela engendre un sentiment de dénuement relatif. Cela pourrait expliquer la courbe plus pentue de la relation entre PIB par habitant et bien-être subjectif dans les pays en développement (voir section 1.1). Nous n’avons pas connaissance d’observations empiriques directes illustrant des comparaisons de revenu à l’échelle mondiale. Seule exception, les travaux de Clark et Senik (2010) qui notent que dans l’enquête européenne précitée, les individus qui n’ont pas accès à Internet sont moins enclins à faire des comparaisons de revenus.

108La vision la plus radicale de l’importance des comparaisons de revenu conduirait à conclure que c’est uniquement parce qu’ils se comparent les uns aux autres que les habitants les plus riches du globe sont plus heureux et les plus pauvres moins heureux. Faut-il en conclure que les pays à faible revenu doivent abandonner toute politique de croissance ? Ce serait là un conseil surprenant. En effet, si les positions relatives sont importantes, beaucoup pourraient trouver étrange de recommander aux pays à bas revenu de conserver leur classement défavorable dans le concert des nations. Même si les comparaisons de revenus engendrent une course vaine entre pays, il n’est pas démontré que le renoncement à la compétition mène au bonheur.

3.2 – Le phénomène d’adaptation

109L’adaptation et « la stagnation hédonique » (« hedonic treadmill ») qui lui est associée constituent l’une des explications classiques du paradoxe d’Easterlin. Les effets de l’habitude détruisent les effets de la croissance sur le bien-être. Cela est dû au rôle délétère des aspirations : « Les aspirations matérielles croissent en proportion du revenu et, de ce fait, on ne s’approche ni ne s’éloigne de la réalisation de ses objectifs matériels, et le bien-être reste inchangé [40] » (Easterlin, 2003).

110L’adaptation est une question centrale en sciences sociales : dans quelle mesure nous habituons-nous à une situation de vie particulière ? Suivant le processus psychologique de l’adaptation, le jugement porté sur une situation présente dépend des expériences passées, ces dernières influençant les attentes (voir Kahneman et Tversky, 1979). Certains psychologues établissent un parallèle entre l’homéostasie qui maintient constante notre température corporelle et l’homéostasie en bien-être subjectif (Cummins, 2003), l’argument étant que cette dernière maintient le bien-être à un niveau spécifique à chaque individu (qui se situerait entre 60 et 80 sur une échelle standardisée de 0-100, avec une valeur moyenne de 75). Ce niveau pourrait être en partie biologiquement déterminé, notamment par des facteurs génétiques. En tout état de cause, l’élément crucial est que, même si des événements positifs et négatifs exercent des effets à court terme sur le bien-être, la plupart des individus retournent à terme à leur niveau de base.

111Quoiqu’initialement partisans de l’hypothèse de l’adaptation, Fujita et Diener (2005) notent qu’au cours des dix-sept années de l’enquête allemande GSOEP, le bien-être d’un quart des individus a changé de manière significative entre les cinq premières années et les cinq dernières années. Diener et al. (2006) proposent cinq révisions importantes de la théorie de l’adaptation hédonique » : 1) le niveau de base des individus n’est pas neutre au plan hédonique ; 2) le niveau de base n’est pas identique chez tous les individus ; 3) une même personne peut avoir de multiples niveaux de base dans les différentes composantes du bien-être (émotions, affects, satisfaction dans la vie) ; 4) les niveaux de base peuvent changer dans certaines conditions, 5) les individus diffèrent par leur degré d’adaptation aux événements.

112Dans le contexte du paradoxe d’Easterlin, c’est l’adaptation au revenu qui nous intéresse plus particulièrement. Concernant l’équation (1) plus haut, il s’agit d’introduire encore une fois un terme supplémentaire dans la fonction d’utilité ; mais cette fois, il ne s’agit pas du revenu d’autrui mais du revenu que l’individu lui-même a connu par le passé. Le bien-être individuel est donc encore le fruit d’une comparaison de revenus mais ici les comparaisons sont internes au sujet, pour utiliser un terme de psychologie. Ceux qui ont gagné plus par le passé sont moins satisfaits de leur niveau actuel de revenu quel qu’il soit.

113En théorie, tout niveau de revenu passé pourrait avoir une incidence négative sur le bien-être actuel, mais en pratique les travaux empiriques ont fait appel au revenu gagné par l’individu un an auparavant (en termes de panel, c’est le revenu que l’individu a déclaré lors de la vague d’enquête précédente, car la plupart des enquêtes auprès des ménages sont réalisées une fois par an).

114

Quel est le lien entre le PIB et la croissance économique ?

115Ce type de fonction d’utilité implique que toute tentative d’accroître le bonheur en augmentant le revenu est vouée à l’échec. Si l’effet (négatif) du revenu passé, via l’accoutumance, est suffisamment puissant, le revenu n’aura pas d’effet durable sur le bien-être au niveau individuel et sociétal.

a – Le phénomène d’adaptation dans les pays développés

116Le travail le plus souvent cité dans le domaine de l’adaptation au revenu est sans doute celui de Brickman et al. (1978), qui montre, à l’aide d’un très petit échantillon (22) de gagnants à la loterie américaine, que ces derniers ne se déclarent pas plus satisfaits de leur vie qu’un groupe de contrôle. Les auteurs y voient le signe de leur adaptation à un revenu plus élevé. Bien que cet article ait été abondamment cité, les résultats ne sont pas réellement aussi univoques. D’une part, les gagnants étaient en réalité plus satisfaits que les autres, mais la faible taille de l’échantillon n’autorisait pas une différence statistiquement significative. D’autre part, l’analyse est faite en coupe transversale et non en panel. De ce fait, il est tout à fait possible que les gagnants à la loterie aient été moins heureux avant de gagner. Ils auraient alors connu un surcroît de bien-être en gagnant à la loterie, mais celui-ci n’aurait pas été détecté par l’analyse.

117Inglehart et Rabier (1986) proposent l’une des premières observations faisant appel à des données explicites sur les changements de revenus ; à partir des données regroupées de l’Eurobaromètre de dix pays d’Europe occidentale entre 1973 et 1983, ils montrent que pour l’essentiel les scores de bien-être ne sont pas corrélés avec le revenu actuel, mais qu’ils sont positivement corrélés avec un changement de situation financière intervenu au cours des douze mois précédents. Ils concluent que les aspirations s’adaptent aux circonstances, si bien que sur le long terme, les caractéristiques stables sont sans effet sur le bien-être.

118Plus récemment, Clark (1999) a examiné la relation entre satisfaction au travail et revenu du travail actuel et passé à partir de deux vagues de données de l’enquête BHPS. Pour les individus qui n’ont pas changé de poste ni d’entreprise, le revenu passé réduit la satisfaction au travail tandis que le revenu actuel l’accroît. Cette observation est cohérente avec une fonction d’utilité qui dépend des variations du revenu. Les données suggèrent une fonction entièrement relative, la satisfaction au travail dépendant uniquement de la variation annuelle du salaire horaire. Plus récemment, Di Tella et al. (2005) et Burchardt (2005) rapportent des résultats reposant respectivement sur les données de panel allemandes et britanniques. Layard et al. recourent aux données du GSOEP pour montrer que l’effet à long terme d’une augmentation du revenu est plus faible que l’effet initial.

119Au lieu d’utiliser le revenu individuel personnel et passé, on peut également recourir au revenu agrégé. Di Tella et al. (2003) examinent le bonheur individuel à l’aide de données couvrant 12 pays européens sur 18 ans et arguent que des « poussées du PIB produisent un accroissement temporaire du bonheur » (p. 817). Autrement dit, c’est la variation et non le niveau du revenu qui affecterait le bien-être.

120Les travaux de l’École de Leyden pré-citée illustrent également le phénomène d’adaptation au revenu : une augmentation de 1 USD du revenu du ménage entraîne une augmentation de 60 % (en deux ans environ) du seuil au-delà duquel les individus jugent leur revenu « excellent », « bon » « suffisant », « mauvais », etc. C’est donc 60 % de l’effet du revenu sur le bien-être qui s’évapore sous l’effet de l’adaptation.

b – L’adaptation dans les pays à bas revenu

121De nombreux travaux sur l’adaptation ont fait appel à des données de panel pour suivre le bien-être individuel à mesure de l’évolution du revenu au cours du temps. S’il existe aujourd’hui une littérature florissante consacrée à l’adaptation considérée sous cet angle dans les pays riches, on observe en même temps une absence quasi-totale d’études dans les pays pauvres, certainement due à l’absence de données de panel dans ces derniers.

122Le travail de Knight et Gunatilaka (2009) fait exception. Il fait appel à une enquête auprès des ménages de la Chine rurale. L’enquête comprend des informations sur la satisfaction dans la vie et relative au revenu, mais aussi sur le revenu minimum que les personnes interrogées jugent nécessaire pour entretenir leur ménage pendant un an. Cette dernière mesure, parfois appelée « question du revenu minimum » (Minimum Income Question), a été introduite par Goedhart et al. (1977). Knight et Gunatilaka considèrent la réponse comme une mesure des aspirations en matière de revenu et observent qu’elle est positivement corrélée au revenu effectif, de sorte que plus les individus gagnent un revenu important, plus le niveau de revenu qu’ils considèrent comme le minimum nécessaire est élevé. Le bien-être subjectif est positivement corrélé avec le revenu individuel, mais négativement corrélé avec le revenu auquel l’individu aspire. Sur ce plan, les résultats sont cohérents avec une adaptation au moins partielle au revenu en Chine [41].

123Barr et Clark (2010), à l’aide de données sud-africaines, analysent le niveau de revenu que les individus déclarent nécessaire pour vivre décemment et pour bien vivre. Une régression économétrique montre que ces deux revenus sont positivement corrélés avec le revenu que l’individu perçoit effectivement et avec celui de son groupe de référence (défini géographiquement). Cette observation est, là encore, cohérente avec un certain degré d’adaptation. Dans le même ordre d’idées, Herrera et al. (2006) établissent une analyse comparative des données d’enquêtes au Pérou et à Madagascar. La satisfaction à l’égard du niveau de vie est corrélée positivement avec le revenu individuel, mais négativement avec le revenu moyen du voisinage et avec le montant minimum que l’individu estime nécessaire pour s’en sortir. À son tour, ce dernier minimum est positivement corrélé avec le revenu individuel effectif, ce qui suggère l’existence d’un effet de cliquet par lequel un revenu élevé accroît les aspirations et réduit la satisfaction.

124Di Tella et MacCulloch (2007, 2010, chapitre 8) apportent des éclairages remarquables basés sur de multiples coupes transversales. Ils découvrent une corrélation positive dans le temps entre revenu par tête et bonheur dans les pays à faible revenu, mais non dans les pays à revenu élevé. Dans ces derniers, le niveau de PIB par habitant atteint en 1960 suffit à expliquer le niveau de bonheur en 2005. Dans les pays à faible revenu, au contraire, le niveau de 1960 et la croissance ultérieure du PIB par habitant ont tous les deux un effet significatif sur le bien-être subjectif de 2005. Les auteurs concluent à la moindre importance de l’adaptation dans les pays à faible revenu : « Les quarante-cinq dernières années de croissance économique (de 1960 à 2005) dans les nations riches du monde n’ont pas apporté de gains de bonheur supérieurs à ceux qui étaient déjà présents une fois atteint le niveau de vie des années 1960. À l’inverse, dans les nations les plus pauvres, nous ne pouvons rejeter l’hypothèse nulle selon laquelle les gains de bonheur enregistrés au cours du dernier demi-siècle de croissance économique ont été identiques aux gains de la croissance enregistrés avant les années 1960. Autrement dit, pour ces nations, c’est encore le niveau absolu (le logarithme) du revenu qui importe pour le bonheur [42]. » (2010, p. 219). Dans les pays riches, le processus d’adaptation réintroduit l’idée d’un effet de seuil dans la relation PIB-bonheur.

3.3 – Échelles bornées : qu’est-ce qui est relatif exactement ?

125Le gain en bien-être associé à une hausse du revenu est-il exclusivement de nature relative (provenant d’une comparaison au revenu des autres ou au revenu passé de l’individu) ? Ou bien est-il possible que les mesures de bonheur elles-mêmes soient relatives (à un contexte implicite) ?

126Il nous semble probable que la satisfaction exprimée sur une échelle ordinale bornée exprime des jugements relatifs, c’est-à-dire la relation entre les réalisations des individus et l’ensemble des possibles (représenté par l’échelle). Van Praag (1991), par exemple, a illustré ce phénomène dans le cadre de situations expérimentales impliquant des échelles bornées : quelle que soit la grandeur concernée, les sujets divisaient toujours la longueur totale de l’échelle en quantiles, et assimilaient ainsi l’échelon le plus élevé au maximum de la valeur envisagée. Si tel est le cas, il n’est pas surprenant qu’une petite minorité de la population seulement choisisse le dixième échelon supérieur sur l’échelle du bonheur, interprétée littéralement comme le nec plus ultra.

127Certes, le fait que l’échelle de bonheur soit interprétée comme une mesure dépendante du contexte est difficile à démêler du fait que le bonheur lui-même dépend du contexte. Cependant, afin d’illustrer la spécificité des échelles bornées, nous avons distingué parmi les indicateurs de qualité de vie (qui sont positivement corrélés avec la croissance), d’une part les mesures car-dinales, qui sont mesurables sur une échelle continue (même si souvent cette dernière n’est pas infinie), telles que l’espérance de vie, le pourcentage de la population alphabétisée, le taux de fécondité des femmes ou le taux brut de scolarisation, et d’autre part les variables ordinales, mesurées sur une échelle bornée, telles que le bonheur, l’indice de démocratie (Polity IV), l’indice des droits de l’Homme ou la confiance [43]. En ne conservant que les pays observés pendant au moins dix ans dans l’enquête World Values Survey et ayant connu un taux de croissance positif, nous avons représenté l’évolution temporelle des grandeurs en question. Les graphiques de la figure 9 représentent séparément ces évolutions dans les pays d’Asie et dans les pays occidentaux de l’OCDE.

128Ces graphiques appellent deux remarques. Premièrement, les mesures objectives mais ordinales et bornées (démocratie, droits humains) tendent à converger vers leur valeur maximale à mesure du développement lié à la croissance du PIB, tandis que les mesures ordinales subjectives (bonheur et confiance) restent inférieures à la borne supérieure de l’échelle. Deuxièmement, les courbes représentant le bonheur moyen, la confiance, les droits de l’homme et la démocratie tendent à être beaucoup plus plates que les courbes des indicateurs cardinaux tels que la fécondité, le taux de scolarisation, l’espérance de vie ou la mortalité infantile, qui affichent des tendances dynamiques beaucoup plus nettes.

129En conclusion, il ne faut pas s’attendre à voir des mesures ordinales sur échelle bornée se comporter sur longue période comme des mesures cardinales. Au lieu d’examiner l’évolution à long terme du niveau moyen de bien-être subjectif (dont l’accroissement est inévitablement limité), il est plus intéressant d’observer la distribution des réponses sur l’échelle proposée. Le fait que la variance du bien-être subjectif tende à chuter à mesure de la croissance du PIB (cf. section 1.b) est très prometteur pour les pays à faible revenu.

Figure 9

Évolution des indices cardinaux et ordinaux de qualité de la vie lors d’un épisode de croissance positive

Quel est le lien entre le PIB et la croissance économique ?

Évolution des indices cardinaux et ordinaux de qualité de la vie lors d’un épisode de croissance positive

4 – Conclusions : Comment utiliser les variables subjectives pour comprendre la relation PIB-bonheur ?

130Les données présentées dans cet article montrent que les variables de satisfaction subjective peuvent être utilisées pour mesurer le bien-être dans les pays en développement. Tout d’abord, ces mesures sont particulièrement adaptées pour saisir le caractère multidimensionnel de la croissance et peuvent être utilisées pour estimer les taux marginaux de substitution entre différents aspects du développement, entre lesquels il peut être impératif d’arbitrer tels qu’une consommation accrue, une plus longue espérance de vie, une dégradation de la qualité de l’air, les embouteillages urbains, etc. Nous disposons ainsi d’un outil utile de politique publique qui vise à maximiser le bien-être à mesure du développement des pays.

131Ensuite, les données subjectives contiennent divers enseignements concernant les gains de bien-être que la croissance peut apporter aux pays en développement. Les données transversales montrent clairement que la croissance du revenu produit des bénéfices tangibles en matière de bonheur et de la satisfaction dans la vie déclarés par les individus, quoique avec des rendements marginaux décroissants (suivant une relation concave). Dans un pays donné, les riches font état d’un niveau de bonheur plus élevé que les pauvres ; de même, ceux qui vivent dans des pays riches sont plus heureux que ceux qui vivent dans des pays pauvres.

132Cependant, les observations sont beaucoup moins tranchées en ce qui concerne les évolutions à long terme du bien-être dans les économies en croissance. La question de savoir si la croissance du PIB augmente le bien-être suscite encore de vifs débats : le coefficient de corrélation est-il négligeable ou non ? Cette question du gain potentiel de la croissance en matière de bien-être est évidemment lourde de conséquences pour les pays en développement.

133Les explications de la faible corrélation entre croissance du revenu et évolution du bien-être subjectif sur longue période font appel à la nature même de la croissance et à la psychologie de l’être humain. Tout d’abord, la croissance peut aller de pair avec des changements qualitatifs non monétaires qui améliorent la « qualité de vie », mais elle peut aussi s’accompagner d’effets secondaires indésirables tels que la pollution, les inégalités de revenu ou le stress au travail. Ensuite, un plus grand pouvoir d’achat accroît le bonheur individuel, mais l’homme est aussi un animal social et les préoccupations relatives (comparaisons de revenus) peuvent réduire l’effet absolu d’une richesse accrue. Cette observation est cohérente avec la relation positive entre revenu et bonheur qui est régulièrement observée au sein des pays, ainsi qu’en comparaison internationale si les comparaisons de revenus sont mondiales et non plus locales. On peut alors adopter une vision très pessimiste et considérer que la croissance est un jeu à somme nulle, dans lequel les riches sont plus heureux que les pauvres, tant au sein d’un pays qu’en comparaison internationale, sans que cela ne permette d’espérer que l’accroissement du revenu pour tous accroisse le bonheur de tous. Cela expliquerait que le bonheur ne semble pas augmenter avec le PIB au cours du temps, sur longue période. Cependant, même si cela était vrai, beaucoup pourraient trouver étrange de recommander aux pays à bas revenu de conserver leur faible classement actuel dans le concert des nations. Tout pays, quel qu’il soit, aura toujours une incitation à progresser dans la hiérarchie. Mais tout gain enregistré par un pays pourrait bien impliquer des pertes pour d’autres pays, si le revenu était uniquement évalué au terme de comparaisons internationales. De même, la croissance du revenu d’une partie de la population au sein d’un pays bénéficiera à cette partie, mais pourrait diminuer le bien-être d’autres groupes.

134Un phénomène analogue est celui de l’adaptation au niveau de vie, au terme de laquelle les individus tendent à revenir à un niveau donné de bien-être. La croissance modifie l’environnement et les aspirations. Si les attentes et les réalisations augmentent au même rythme, les individus ne se sentiront pas plus heureux. S’ils ne prennent pas conscience du fait que leurs attentes et leurs réalisations tendent à évoluer de concert, les individus aspireront à être plus riches, mais ils ne se trouveront pas plus heureux lorsque leurs espoirs se réaliseront. Ce mouvement perpétuel et vain peut être vu comme illusoire, à la suite d’Easterlin, mais il peut également être interprété comme un mécanisme issu de l’évolution, et assurant la permanence des efforts de progrès chez les êtres humains (Rayo et Becker, 2007).

135L’une des questions cruciales de cette littérature est l’importance comparée des préoccupations de revenu absolu et de revenu relatif. L’utilité du revenu est-elle entièrement relative ? Et la proportion relatif/absolu est-elle la même dans les pays en développement et dans les pays développés ? Les données empiriques sur l’étendue des comparaisons de revenus sont beaucoup plus rares dans les pays en développement. Les données dont nous disposons pour l’instant sont porteuses de deux enseignements importants : les comparaisons de revenus semblent bien avoir une incidence sur le bien-être subjectif, même dans les pays très pauvres ; cependant, l’adaptation pourrait être un phénomène propre aux pays riches.

136Enfin, croissance et développement ne sont pas seulement une affaire d’augmentation quantitative de la consommation, de la production et de l’accumulation de capital. Ils impliquent également la transformation qualitative de la gouvernance politique et le développement du marché. Ces processus qualitatifs et quantitatifs connaissent des phases de décollage et de seuils. Le changement de régime est une dimension importante de ces variations non linéaires ; or, il est évident que ces dernières sont fidèlement reflétées par les mesures de satisfaction subjective. Le cas des pays en transition est remarquable à cet égard : les scores moyens de satisfaction dans la vie suivent étroitement les fluctuations du PIB pendant les dix premières années du processus de transition, jusqu’à ce que le régime se stabilise. En revanche, dans les régimes stables tels que la France, on n’observe plus de relation entre la croissance du PIB et l’évolution de la satisfaction dans la vie, sur le long terme. Notre interprétation est que dès qu’il est stabilisé, le régime économique devient le cadre de référence de la population.

137S’il est difficile de constater d’importants gains de bien-être subjectif au cours d’épisodes longs de croissance économique, il est néanmoins possible d’établir un constat intéressant concernant le niveau et la répartition du bien-être subjectif en fonction du niveau de développement des pays. Les faits stylisés sont les suivants : (i) le bien-être subjectif augmente avec le PIB par habitant mais (ii) l’écart-type du bien-être subjectif diminue avec le PIB par habitant. De ce fait, (iii) il y a une forte relation négative entre la valeur moyenne et l’écart-type du bien-être subjectif au sein d’un pays. Autrement dit, la croissance du PIB réduit les inégalités du bien-être subjectif. Ceci constitue certainement un résultat souhaitable. Derrière le voile de l’ignorance, des individus averses au risque préféreront une société dans laquelle le bien-être est plus également réparti, toutes choses égales par ailleurs.

138Le recours aux mesures subjectives de bien-être est particulièrement intéressant pour évaluer les phénomènes sociaux qui ne sont pas mesurables par la méthode standard des préférences révélées. Ainsi, dès lors que des interactions sociales, des préférences sociales ou des externalités sont en jeu, il devient plus difficile d’établir le lien entre les préférences individuelles et les actions individuelles (on ne peut pas acheter moins d’inflation, de chômage ou d’inégalités de revenu par une transaction sur un marché). Il convient cependant d’utiliser les variables subjectives en complément - et non en substitut - des préférences révélées par l’action. Lorsque les individus « votent avec leurs pieds » par exemple, il est difficile de dénier toute importance à leurs actes au prétexte que le message n’est pas confirmé par les données subjectives. Ainsi, en ce qui concerne la relation entre croissance et bien-être, tant que les migrations internationales resteront clairement uni-directionnelles, des pays à bas revenu vers les pays à revenu élevé, il semblera extrêmement difficile d’arguer que la croissance du PIB n’apporte pas de gain de bien-être. Les préférences révélées par les migrations internationales cadrent avec la corrélation positive entre revenu et bien-être mise au jour par les données individuelles en coupes transversales instantanées.

139Notre position est que les analyses fondées sur des données dynamiques agrégées de bien-être subjectif sont beaucoup moins robustes et fiables que les analyses sur données individuelles (en coupes instantanées ou de panel). Les comparaisons longitudinales entre pays reposent sur des mesures agrégées qui, par définition, présentent une moindre variance et sont moins puissantes en termes d’inférence statistique. De plus, il est possible que les jugements de satisfaction exprimés sur une échelle bornée produisent intrinsèquement des jugements relatifs à l’ensemble des possibles (représentés par l’échelle bornée). Dans ce cas, il faut s’attendre à ce qu’une petite minorité seulement d’individus choisissent le 10e degré de l’échelle, interprété comme le nec plus ultra. De fait, les variables cardinales telles que la fécondité, l’espérance de vie ou l’alphabétisation présentent des tendances dynamiques bien plus nettes que les variables qualitatives et ordinales, mesurées sur une échelle bornée, telles que les indicateurs de gouvernance, de liberté ou de bonheur.

140Afin d’éclairer les vifs débats qui entourent la relation entre croissance du revenu et bien-être, il est souhaitable qu’à l’avenir les chercheurs puissent disposer d’enquêtes de panel dans les pays à faible revenu.

Annexe 1

Tableau A1

Statistiques descriptives des variables prises dans la base de données WVS

Statistiques descriptives des variables prises dans la base de données WVS

Description des variables

141Toutes les variables sont accessibles dans la banque de données : http://www.worldvaluessurvey.org/.

142Bonheur : « Si vous deviez réfléchir à votre vie en général en ce moment, diriez-vous que vous êtes globalement ? » : 1. Pas du tout heureux ; 2 ; Pas très heureux ; 3. Assez heureux ; 4. Très heureux.

143Satisfaction dans la vie :« Tout bien considéré, êtes-vous satisfait de votre vie en ce moment ? » : 1 (pas du tout satisfait)… 10 (très satisfait) ». Réponses proposées de 1 (pas satisfait) à 10 (très satisfait).

144Confiance : « De manière générale, diriez-vous qu’on peut faire confiance à la plupart des gens ou qu’on n’est jamais trop prudent? », Réponses proposées : 1. On peut faire confiance à la plupart des gens ; 0. On n’est jamais trop prudent.

145Taux de fécondité : nombre d’enfants que mettrait au monde une femme qui vivrait jusqu’à la fin de ses années de procréation et qui aurait un nombre d’enfants correspondant aux taux de fécondité par âge actuels.

146Croissance du PIB : pourcentage annuel de croissance du PIB aux prix du marché en monnaie locale constante. Les agrégats sont calculés en USD constants de 2000.

147PIB par habitant en dollars de 2000 : le PIB par habitant est le produit intérieur brut divisé par la population en milieu d’année. Les données sont exprimées en USD constants.

148Indice de Gini : l’indice de Gini est la mesure dans laquelle la répartition du revenu (ou, dans certains cas, des dépenses de consommation) entre les individus ou ménages d’une économie s’écarte d’une distribution parfaitement égale. Une courbe de Lorenz représente les pourcentages cumulés du revenu total reçu par rapport au nombre cumulé de bénéficiaires, en commençant par l’individu ou le ménage le plus pauvre. L’indice de Gini mesure l’aire entre la courbe de Lorenz et une ligne hypothétique d’égalité parfaite, exprimée en pourcentage de l’aire maximale située sous la ligne. Ainsi, un indice de Gini de 0 représente une égalité parfaite, tandis qu’un indice de 100 représente une inégalité parfaite.

149Espérance de vie à la naissance : nombre d’années qu’un nouveau-né vivrait si les règles générales de mortalité au moment de sa naissance devaient rester les mêmes tout au long de sa vie.

150Taux de mortalité infantile (pour 1 000 enfants de moins d’un an) : nombre d’enfants qui décèdent avant d’atteindre l’âge d’un an pour 1 000 naissances vivantes au cours d’une année donnée.

151Taux brut de scolarisation en % : scolarisation dans l’enseignement primaire, secondaire et supérieur.

152Alphabétisation des adultes : taux en pourcentage.

153Freedom House (http://www.freedomhouse.org) : droits politiques permettant aux individus de participer librement au processus politique, comprenant le droit de voter librement pour des solutions différentes dans le cadre d’élections légitimes, de se porter candidat à des élections, d’adhérer à des partis et organisations politiques, et d’élire des représentants qui ont un impact décisif sur les politiques publiques et sont responsables devant les électeurs. La liste précise des droits considérés varie d’une année à l’autre. Les pays sont notés sur une échelle de 1 (les plus libres) à 7 (les moins libres).

154Démocratie : Moyenne de Freedom House et Polity, transformée sur une échelle de 0-10, où 0 est le moins démocratique et 10 le plus démocratique (http://www.govindicators.org ).

Annexe 2

La relation entre revenu et bonheur : sources et estimations, résumé. Mesures subjectives de bien-être

155

  • Bonheur : « Si vous deviez réfléchir à votre vie en général en ce moment, diriez-vous que vous êtes globalement ? : 1. Pas du tout heureux ; 2 ; Pas très heureux ; 3. Assez heureux ; 4. Très heureux.
  • Satisfaction dans la vie : Tout bien considéré, êtes-vous satisfait de votre vie en ce moment ? : 1 (pas du tout satisfait)… 10 (très satisfait).

1 – Relation statistique entre revenu individuel et bonheur individuel

156Consensus : revenu plus élevé ? bonheur plus élevé. Dans un pays, les individus riches sont plus heureux que les pauvres.

157Enquêtes auprès de ménages représentatifs de la population. Estimations transversales nationales.

Pays occidentaux développés

158German Socio-Economic Panel (GSOEP), British Household Panel Survey (BHPS), enquête suisse auprès des ménages, enquête australienne auprès des ménages (HILDA), General Social Survey (États-Unis), enquête japonaise auprès des ménages, Pays-Bas, Danemark, etc.

159European Values Survey (EVS), European Social Survey (ESS), Eurobaromètre.

Pays en transition

160Albanie, Bulgarie, Lettonie, Roumanie, Russie, Estonie, Lituanie, Hongrie, Bélarus, Pologne, Ukraine, etc.

161Life in Transition Survey (LITS, 2006), European Social Survey, European Values Survey.

Enquêtes auprès des ménages asiatiques

162Chine, Inde, Shanghai

Enquêtes auprès des ménages d’Afrique et du Moyen-Orient

163Argentine, Brésil, Chili, Éthiopie, Mexique, Nigeria, Pérou, Corée du Sud, Afrique du Sud (SALDRU), Tanzanie, Turquie, Venezuela

Enquêtes internationales

164World Values Survey (WVS, 1981- 2008, 5 cycles, 105 pays).

165International Social Survey Program (ISSP, 101 pays)

166Gallup World Poll (2006, 105 pays).

167Baromètre latino-américain (18 pays)

168European Social Survey (25 pays)

169European Values Survey

2 – Relation dynamique entre le revenu individuel et le bonheur individuel

170Estimations nationales. Analyse des données de panels au niveau individuel ;

171Consensus : revenu plus élevé ? bonheur plus élevé. Les individus deviennent plus heureux à mesure qu’ils s’enrichissent.

Données de panels individuels dans les pays développés

172GSOEP, BHPS, HILDA, Pays-Bas, Danemark

Données de panels individuels dans les pays à faible développement

173RLMS (Russie), ULMS (Ukraine), Pérou, LSMS (Tadjikistan)

3 – Relation statistique instantanée entre le revenu national et le bonheur moyen

174Mesures agrégées, estimations transnationales.

175Consensus : revenu plus élevé ? bonheur plus élevé. Les individus qui vivent dans des pays riches sont plus heureux que ceux qui vivent dans des pays pauvres.

4 – Relation dynamique entre le revenu national et le bonheur moyen

176Mesures agrégées, estimations transnationales.

177Pas de consensus. Constats divergents.

178? La croissance du revenu n’accroît pas le bonheur dans le temps.

  • Easterlin (2005a), Easterlin et Sawangfa (2005, 2009), Easterlin et Angelescu (2007), Easterlin (2009)
  • Layard, Brockmann, Delhey, Welzel, Yuan (2009)
? La croissance du revenu accroît le bonheur dans le temps
  • Stevenson et Wolfers (2008)
  • Deaton (2008), Gallup (2006)
  • Helliwell (2002)
  • Blanchflower (2008)
? La croissance du revenu accroît le bonheur dans le temps, mais pas toujours et faiblement
  • Hagerty et Veenhoven (2000, 2003, 2006), WVS (coefficient positif et statistiquement significatif, mais pas dans tous les pays).
  • Inglehart, Peterson et Welzel (2008): WVS, BHPS, GSS (coefficient positif et statistiquement significatif, mais pas dans tous les pays), Kenny (2005), idem.
  • Layard, Mayraz et Nickell (2010) : coefficient positif mais pas toujours statistiquement significatif.
  • Oswald (1997) : dans les séries du GSS et de l’Eurobaromètre, coefficient positif mais pas toujours statistiquement significatif.
  • Di Tella et MacCulloch (2008) : coefficient positif mais significativité statistique faible.

Notes

  • [*]

    , PSE, ENS, 48 bd Jourdan, 75014 Paris, France. Nous remercions Hélène Blake pour son aide précieuse dans nos recherches et le CEPREMAP pour son soutien financier. Nous sommes extrêmement reconnaissants à Emmanuel Commolet, Nicolas Gury et Oliver Charnoz pour leurs commentaires sur une version précédente, et à Alpaslan Akay, Alexandru Cojocaru, Luca Corazzini, Ada Ferrer-i-Carbonell, Armin Falk, Carol Graham, Olof Johansson-Stenman, John Knight, Andrew Oswald, Bernard Van Praag, Hillel Rapoport, Mariano Rojas, Russell Smyth, et Oded Stark pour leurs conseils.

  • [1]

    « If you were to consider your life in general these days, how happy or unhappy would you say you are, on the whole? : 1. Not at all happy; 2. Not very happy ; 3. Fairly happy ; 4. Very happy. »

  • [2]

    « All things considered, how satisfied are you with your life as a whole these days? : 1(dissatisfied) … 10 (very satisfied) ».

  • [3]

    « How dissatisfied or satisfied are you with your life overall? »

  • [4]

    « Not satisfied at all », « Neither satisfied nor dissatisfied », « Completely satisfied ».

  • [5]

    « Taken all together, how would you say things are these days, would you say that you are very happy, pretty happy, or not too happy? ».

  • [6]

    Voir, par exemple, les constats de Di Tella, MacCulloch et Oswald (2003), basés sur l’analyse du bien-être déclaré par 250 000 Européens et Américains sélectionnés de manière aléatoire des années 1970 aux années 1990.

  • [7]

    « As far as I am aware, in every representative national survey ever done, a significant bivariate relationship between happiness and income has been found. »

  • [8]

    « When we plot average happiness versus average income for clusters of people in a given country at a given time…rich people are in fact a lot happier than poor people. It’s actually an astonishingly large difference. There’s no one single change you can imagine that would make your life improve on the happiness scale as much as to move from the bottom 5 percent on the income scale to the top 5 percent. »

  • [9]

    « Overall, the average well-being-income gradient is 0.38, with the majority of the estimates between .25 and .45 and 90 percent are between 0.07 and 0.72. In turn, much of the heterogeneity likely reflects simple sampling variation: the average country-specific standard error is 0.07, and 90 percent of the country-specific regressions have standard errors between 0.04 and 0.11. »

  • [10]

    « A between-country well-being-GDP gradient [..] typically centered around 0.4. »

  • [11]

    Ces estimations varient en raison de la composition de l’échantillon et des contrôles intégrés aux régressions.

  • [12]

    « Each Doubling of GDP is Associated with a Constant Increase in Life Satisfaction ».

  • [13]

    Selon Guriev et Zhuravskaya (2009), le moindre niveau de bonheur dans les pays en transition s’explique par la dégradation de la fourniture de biens publics, l’accroissement de la volatilité macroéconomique et l’inadéquation du capital humain des résidents formés avant la transition (chômage).

  • [14]

    « Once a country has over $15,000 per head, its level of happiness appears to be independent of its income per head. »

  • [15]

    Cette notion de point de satiété remonte au concept d’Adam Smith du « dernier degré de richesse » (« full complement of riches »), au-delà duquel on ne pourrait plus souhaiter davantage d’argent. Les grands propriétaires terriens du xviiie siècle avaient, selon lui, atteint cette limite. Cependant, il peut y avoir une limite à la quantité de richesse qu’une personne peut posséder dans une société donnée à un moment donné, mais il ne s’ensuit pas que cette limite ne puisse être repoussée grâce aux nouveaux choix permis par la croissance économique (Internet par exemple). Autrement dit, le « dernier degré de richesse » pourrait être plus important dans les pays riches que dans les pays moins développés.

  • [16]

    « if we compare countries, there is no evidence that richer countries are happier than poorer ones — so long as we confine ourselves to countries with incomes over $15,000 per head.… At income levels below $15,000 per head things are different… »

  • [17]

    « income provides happiness at low levels of development but once a threshold (around $10,000) is reached, the average income level in a country has little effect on average subjective well-being. »

  • [18]

    « The transition from a society of starvation to a society of security brings a dramatic increase in subjective well-being. But we find a threshold at which economic growth no longer seems to increase subjective well being significantly. This may be linked with the fact that, at this level, starvation is no longer a real concern for most people. Survival begins to be taken for granted […] At low levels of economic development, even modest economic gains bring a high return in terms of caloric intake, clothing, shelter, medical care and ultimately in life expectancy itself. […]. But once a society has reached a certain threshold of development … one reaches a point at which further economic growth brings only minimal gains in both life expectancy and subjective well-being. There is still a good deal of cross national variation, but from this point on non-economic aspects of life become increasingly important influences on how long and how well people live. »

  • [19]

    « Happiness and life satisfaction rise steeply as one moves from subsistence-level poverty to a modest level of economic security and then levels off. Among the richest societies, further increases in income are only weakly linked with higher levels of SWB. »

  • [20]

    « the well-being-GDP gradient is about twice as steep for poor countries as for rich countries. That is […] a rise in income of $100 is associated with a rise in well-being for poor countries that is about twice as large as for rich countries. »

  • [21]

    « The point estimates are, on average, about three times as large for those countries with incomes above $15,000 compared to those countries with incomes below $15,000”. […] Taken at face value, the Gallup results suggest that a 1 percent rise in GDP per capita would have about three times as large an effect on measured well-being in rich as in poor nations. Of course, a 1 percent rise in U.S. GDP per capita is about ten times as large as a 1 percent rise in Jamaican GDP per capita. »

  • [22]

    « the relationship between log per capita income and life satisfaction is close to linear. The coefficient is 0.838, with a small standard error. A quadratic term in the log of income has a positive coefficient: confirming that the slope is higher in the richer countries! […] Using 12000$ of income per capita as a threshold between rich and poor countries shows that the slope in the higher income countries is higher! […] If there is any evidence for a deviation, it is small and is probably in the direction of the slope being higher in the high-income countries. »

  • [23]

    « the slope is steepest among the poorest countries, where the income gains are associated with the largest increases in life satisfaction, but it remains positive and substantial even among the rich countries; it is not true that there is some critical level of GDP per capita above which income has no further effect on life satisfaction. »

  • [24]

    Il convient de souligner que si la fonction log est bien concave, elle n’a pas de borne supérieure. y=log(x), y ne tend pas vers 1 lorsque x tend vers l’infini. Pourtant, c’est le message qu’une grande majorité de spécialistes du domaine ont tiré de l’utilité marginale décroissante du revenu et de la bonne correspondance entre la forme fonctionnelle log-linéaire et la relation entre revenu et bonheur.

  • [25]

    « Rather than diminishing marginal utility of income, there is a zero marginal utility of income. »

  • [26]

    « The usual constancy of subjective well-being in the face of rising GDP per capita has typically been reconciled with the cross-sectional evidence on the grounds that the time series observations for developed nations correspond to the upper income range of the cross-sectional studies, where happiness changes little or not at all as real income rises.” … “the income change over time within the income range used in the point-of-time studies do not generate the change in happiness implied by the cross-sectional pattern. »

  • [27]

    « in 1972, the cohort of 1941-1950 had a mean per capita income of about 12000$ (expressed in 1994 constant prices). By the year 2000, the cohort’s average income had more than doubled, rising to almost 27000$. According to the cross-sectional relation, this increase should have raised the cohort’s mean happiness from 2.17 to 2.27. In reality, the actual happiness of the cohort did not change. »

  • [28]

    « knowing the actual change over time in a country’s GDP per capita and the multi-country cross-sectional relation of SWB to GDP per capita adds nothing, on average, to one’s ability to predict the actual time-series change in SWB in a country. »

  • [29]

    « In many cases, the results contradict the assumption that, despite economic growth, and other changes, the publics of given societies have not gotten any happier. They show that the American and British series show a downward trend in happiness from 1946 to 1980, but an upward trend thereafter. »

  • [30]

    « In general, among the countries for which we have a long-term data, 19 out of the 26 countries show rising happiness levels. In several of these countries- India, Ireland, Mexico, Puerto Rico and South Korea- there are steeply rising trends. The other countries with rising trends are Argentina, Canada, China, Denmark, Finland, France, Italy Japan, Luxembourg, the Netherlands, Poland, South Africa, Spain and Sweden. Three countries (the U.S., Switzerland and Norway) show flat trends from the earliest to the latest survey. Only four countries (Austria, Belgium, the U.K and West Germany) show downward trends. »

  • [31]

    « the standard deviation in GDP per capita in the cross section from Diener and Oishi was about 8000$, whereas the standard deviation in Hagerty time-series (for the same countries) was only about ¼ of that (2000$) […] within a country in 25 years. »

  • [32]

    « confusing a short-term positive happiness-income association, due to fluctuations in macroeconomic conditions, with the long-term relationship. We suggest, speculatively, that this disparity between the short and long-term association is due to the social psychological phenomenon of “loss aversion”. »

  • [33]

    « A revolutionary enlargement of freedom for women. »

  • [34]

    « industrialization, urbanization, high educational standards and a steady increase in the overall wealth of society [are] basic conditions sustaining democracy. »

  • [35]

    « rapid and sustained rise in real output per head and attendant shifts in production technology, factor input requirements, and the resource allocation of a nation. »

  • [36]

    « If social and political indicators of QoL are, at present, positively associated with GDP per capita, it is often because the countries that first implemented the new production technology underlying modern economic growth were also the first to introduce, often via public policy, new advances in knowledge in the social and political realms. »

  • [37]

    « The long-run income elasticity of infant and child mortality in developing countries lies between 0.2 and 0.4”. … “over a half a million child deaths in the developing world in 1990 alone can be attributed to the poor economic performance in the 1980s. »

  • [38]

    De sorte qu’un revenu du voisinage plus élevé est associé à une plus grande satisfaction. Cela rejoint les constats de Clark et al (2009) sur les données relatives à de petits quartiers au Danemark.

  • [39]

    « How important is it to you to compare your income with other people’s incomes? ».

  • [40]

    « Material aspirations increase commensurately with income, and as a result, one gets no nearer to or farther away from the attainment of one’s material goals, and well-being is unchanged. »

  • [41]

    Le travail de Castillo (2010) mentionné plus haut montre également que la satisfaction à l’égard du revenu au Mexique est positivement corrélée avec l’évaluation que fait la personne interrogée de son revenu individuel actuel par rapport à ses aspirations. Si les aspirations augmentent avec le revenu individuel, c’est également cohérent avec la thèse de l’adaptation.

  • [42]

    « The past 45 years of economic growth (from 1960 to 2005) in the rich nations of the world have not brought happiness gains above those that were already in place once the 1960s standard of living had been achieved. However, in the poorest nations, we cannot reject the null hypothesis that the happiness gains they experienced from the past half century of economic growth have been the same as the gains from growth prior to the 1960s. In other words, for these nations, it is still the absolute level of (the logarithm of) income that matters for happiness. »

  • [43]

    Question disponible dans l’enquête WVS : « De manière générale, diriez-vous qu’on peut faire confiance à la plupart des gens ou qu’on n’est jamais trop prudent : 1. On peut faire confiance à la plupart des gens ; 0. On n’est jamais trop prudent ».

Références

  • AKAY, A., and MARTINSSON, P. (2008). “Does Relative Income Matter for the Very Poor? Evidence from Rural Ethiopia”. IZA, Discussion Paper No. 3812.
  • AKAY, A., MARTINSSON, P., and MEDHIN, H. (2009). “ “Does Positional Concern Matter in Poor Societies? Evidence from a Survey Experiment in Rural Ethiopia”. IZA, Discussion Paper No. 4354.
  • ALESINA, A. and GIULIANO, P. (2008). Preferences for Redistribution. Harvard University and UCLA mimeo.
  • En ligneALPIZAR, F., CARLSSON, F., and JOHANSSON-STENMAN, O. (2005). “How much do we care about absolute versus relative income and consumption?”. Journal of Economic Behavior and Organization,56, 405-421.
  • ANDERSSON, F. (2006). “Is Concern for Relative Consumption a Function of Relative Consumption?”. Göteborg University, mimeo.
  • En ligneAPPLETON, S., and SONG, L. (2008). “Life Satisfaction in Urban China: Components and Determinants”. World Development,36, 2325-2340.
  • En ligneBARR, A. and CLARK, D. (2010). “Do the Poor Adapt to Low Income, Minimal Education and Ill-Health?”, Journal of African Economies, 19, 257-293.
  • En ligneBECKER, G. S., PHILIPSON, T.J. and SOARES, R.R. (2005). “The quantity and quality of life and the evolution of world inequality”. American Economic Review, 95, 277-291.
  • En ligneBLANCHFLOWER D.G. (2008). “International evidence on well-being”. IZA Discussion Paper No. 3354.
  • En ligneBLANCHFLOWER D.G. and OSWALD A. (2004). “Well-being over time in Britain and the USA”. Journal of Public Economics, 88, 1359-1386.
  • En ligneBLANCHFLOWER D.G. and OSWALD A. (2005). “Happiness and the Human development index: the paradox of Australia”. Australian Economic Review, 38, 307-318.
  • En ligneBOOKWALTER, J., and DALENBERG, D. (2010). “Relative to What or Whom? The Importance of Norms and Relative Standing to Well-Being in South Africa”. World Development,38, 345-355.
  • En ligneBRICKMAN, P., D., C., and JANOFF-BULMAN, R. (1978). “Lottery winners and accident victims: Is happiness relative?”. Journal of Personality and Social Psychology,36, 917-927.
  • En ligneBROCKMANN, H., DELHEY, J., WELZEL, C., and YUAN, H. (2009). “The China Puzzle: Falling Happiness in a Rising Economy”. Journal of Happiness Studies,10, 387-405.
  • BROWN, G., GARDNER, J., OSWALD, A.J., and QIAN, J. (2008). “Does Wage Rank Affect Employees’ Wellbeing?”. Industrial Relations,47, 355 - 389.
  • BROWN, P., BULTE, E. and ZHANG, X. (2010). “Positional Spending and Status Seeking in Rural China.” Journal of Development Economics, à paraître.
  • En ligneBURCHARDT, T. (2005). “Are One Man’s Rags Another Man’s Riches? Identifying Adaptive Preferences Using Panel Data”. Social Indicators Research,74, 57-102.
  • CARD, D., MAS, A., MORETTI, E. and SAEZ, E. (2010). “Inequality at Work: The Effect of Peer Salaries on Job Satisfaction”, NBER Working Paper No. 16396.
  • En ligneCARLSSON, F., GUPTA, G., and JOHANSSON-STENMAN, O. (2009). “Keeping up with the Vaishyas Caste and relative standing in India”. Oxford Economic Papers,61, 52-73.
  • CASTILLA, C. (2010). “Subjective poverty and reference dependence”. UNU-WIDER Working Paper No. 2010/76.
  • En ligneCLARK, A.E. (1996). “L’utilité est-elle relative? Analyse à l’aide de données sur les ménages”. Economie et Prévision,121, 151-164.
  • En ligneCLARK, A.E. (1999). “Are Wages Habit-Forming? Evidence from Micro Data”. Journal of Economic Behavior and Organization,39, 179-200.
  • En ligneCLARK, A.E. (2001). “What Really Matters in a Job? Hedonic Measurement Using Quit Data”. Labour Economics,8, 223-242.
  • En ligneCLARK, A.E. (2003). “Unemployment as a Social Norm: Psychological Evidence from Panel Data”. Journal of Labor Economics,21, 323-351.
  • En ligneCLARK, A.E., FRIJTERS, P., and SHIELDS, M. (2008). “Relative Income, Happiness and Utility: An Explanation for the Easterlin Paradox and Other Puzzles”. Journal of Economic Literature,46, 95-144.
  • CLARK, A.E., GEORGELLIS, Y. and SANFEY, P. (1998). “Job Satisfaction, Wage Changes and Quits: Evidence from Germany”. Research in Labor Economics,17, 95-121.
  • CLARK, A.E. and FISCHER, J.A.V. (2009). in OECD, 2009. Subjective well-being and social policy.
  • En ligneCLARK, A.E., KNABE, A., and RÄTZEL, S. (2010). “Boon or Bane? Others’ Unemployment, Well-being and Job Insecurity”. Labour Economics,17, 52-61.
  • En ligneCLARK, A.E., KRISTENSEN, N., and WESTERGÅRD-NIELSEN, N. (2009). “Economic Satisfaction and Income Rank in Small Neighbourhoods”. Journal of the European Economic Association,7, 519-527.
  • En ligneCLARK, A.E., and OSWALD, A.J. (1996). “Satisfaction and Comparison Income”. Journal of Public Economics,61, 359-81.
  • En ligneCLARK, A.E., and SENIK, C. (2010). “Who compares to whom? The anatomy of income comparisons in Europe”. Economic Journal,120, 573-594.
  • En ligneCOHEN, S., DOYLE, W., TURNER, R., ALPER, C., and SKONER, D. (2003). “Emotional style and susceptibility to the common cold”. Psychosomatic Medicine,65, 652-657.
  • COJOCARU, A. (2010). “On relative status inference when reference groups are unobserved: a cautionary tale”. University of Maryland, mimeo.
  • CORAZZINI L., ESPOSITO L., MAJORANO F. “Reign in hell or serve in heaven? A cross-country journey into the relative vs absolute perceptions of well being”. University of Padua, mimeo.
  • En ligneCUMMINS, R. (2003). “Normative life satisfaction: measurement issues and a homeostatic model”. Social Indicators Research,64, 225-256.
  • En ligneDANNER, D., SNOWDON, D., and FRIESEN, W. (2001). “Positive Emotions in Early Life and Longevity: Findings from the Nun Study”. Journal of Personality and Social Psychology,80, 804-813.
  • DAVIDSON, R. (2004). “Well-being and affective style: neural substrates and biobehavioural correlates”. Philosophical Transactions of the Royal Society, London B,359, 1395–1411.
  • En ligneDEATON, A. (2008). “Income, Health and Well-Being around the World: Evidence from the Gallup World Poll”. Journal of Economic Perspectives,22, 53-72.
  • En ligneDI TELLA, R., HAISKEN-DE NEW, J. and MACCULLOCH, R. (2007). “Happiness Adaptation to Income and to Status”. Harvard Business School, mimeo.
  • En ligneDI TELLA, R., and MACCULLOCH, R. (2006). “Some Uses of Happiness Data in Economics”. Journal of Economic Perspectives,20, 25-46.
  • En ligneDI TELLA, R., and MACCULLOCH, R. (2008). “Gross National Happiness as an Answer to the Easterlin Paradox?”. Journal of Development Economics,86, 22-42.
  • En ligneDI TELLA, R., MACCULLOCH, R.J., and OSWALD, A.J. (2001). “Preferences over Inflation and Unemployment: Evidence from Surveys of Happiness”. American Economic Review,91, 335-341.
  • En ligneDI TELLA, R., MACCULLOCH, R.J., and OSWALD, A.J. (2003). “The Macroeconomics of Happiness”. Review of Economics and Statistics,85, 809-827.
  • DIENER E., HELLIWELL J. and KAHNEMAN D. eds. (2009). International Differences in Wellbeing, Princeton: Princeton University Press.
  • DIENER, E. and LUCAS, R.E. (1999). Personality and subjective well-being. In Kahneman, D., Diener, E. and Schwarz, N. (Eds), Foundations of Hedonic Psychology: Scientific Perspectives on Enjoyment and Suffering, Chapter 11. New York: Russell Sage Foundation.
  • En ligneDIENER, E., LUCAS, R., and SCOLLON, C. (2006). “Beyond the Hedonic Treadmill”. American Psychologist,61, 305-314.
  • En ligneEASTERLIN, R. (1974). Does Economic Growth Improve the Human Lot? Nations and Households in Economic Growth. P. A. David and W. B. Melvin. Palo Alto, Stanford University Press: 89-125.
  • En ligneEASTERLIN, R.A. (1995). “Will Raising the Incomes of All Increase the Happiness of All?”. Journal of Economic Behavior and Organization,27, 35-47.
  • En ligneEASTERLIN R. (2000). “The worldwide standard of living since 1800”. Journal of Economic Perspectives, 14, 7-26
  • En ligneEASTERLIN, R.A. (2001). “Income and Happiness: Towards a Unified Theory”. Economic Journal,111, 465-484.
  • En ligneEASTERLIN, R. (2003). “Explaining Happiness”. Proceedings of the National Academy of Science,100, 11176-11183.
  • En ligneEASTERLIN, R. (2005a). “Diminishing Marginal Utility of Income? Caveat Emptor.” Social Indicators Research70, 243-255.
  • EASTERLIN, R. (2005b). “Feeding the Illusion of Growth and Happiness: A Reply to Hagerty and Veenhoven”. Social Indicators Research,74, 429-443.
  • EASTERLIN R. and ANGELESCU L. (2009). “Happiness and growth the world over: time series evidence on the Happiness-income paradox”, IZA Discussion Paper No. 4060.
  • EASTERLIN R. and ANGELESCU L. (2010). “Modern Economic Growth: Cross Sectional and Time Series Evidence” in Kenneth C. Land, ed., Handbook of Social Indicators and Quality of Life Research, à paraître, New York and London: Springer.
  • En ligneEASTERLIN R. and SAWANGFA O. (2010). “Happiness and Growth: Does the Cross Section Predict Time Trends? Evidence from Developing Countries”, in E. Diener, J. Helliwell, and D. Kahneman, eds. International Differences in Well-Being. Princeton, NJ., Princeton University Press, chapter 7, pp. 162-212.
  • En ligneEASTERLIN R. and ZIMMERMANN A. (2006). “Life satisfaction and economic outcomes in Germany in pre and post unification”. IZA Discussion Paper No. 2494.
  • En ligneEASTERLIN R. and ZIMMERMANN A. (2009). “Lost in Transition: Life Satisfaction on the Road to Capitalism,” Journal of Economic Behavior and Organization, 71, 130-145.
  • En ligneFAFCHAMPS, M. and SHILPI, F. (2008). “Subjective welfare, isolation, and relative consumption,” Journal of Development Economics, 86, 43-60.
  • En ligneFERRER-I-CARBONELL, A. (2005). “Income and well-being: an empirical analysis of the comparison income effect”. Journal of Public Economics,89, 997-1019.
  • En ligneFERRER-I-CARBONELL, A., and FRIJTERS, P. (2004). “How important is methodology for the estimates of the determinants of happiness?”. Economic Journal,114, 641-659.
  • En ligneFLIEßBACH, K., WEBER, B., TRAUTNER, P., DOHMEN, T., SUNDE, U., ELGER, C., and FALK, A. (2007). “Social comparison affects reward-related brain activity in the human ventral striatum”. Science,318, 1305-1308.
  • FREEMAN, R. (1978). Job satisfaction as an economic variable. American Economic Review, vol. 68, pp. 135-141.
  • En ligneFREY, B.S., and STUTZER, A. (2000). “Happiness, Economy and Institutions”. Economic Journal,110, 918-938.
  • En ligneFREY, B., and STUTZER, A. (2007). “Should National Happiness be Maximized?”. University of Zurich, mimeo.
  • En ligneFUJITA, F., and DIENER, E. (2005). “Life Satisfaction Set Point: Stability and Change”. Journal of Personality and Social Psychology,88, 158-164.
  • En ligneGAO, W., and SMYTH, R. (2010). “Job satisfaction and relative income in economic transition: status or signal? The case of urban China”. China Economic Review,21, 442-455.
  • En ligneGARDNER, J., and OSWALD, A.J. (2006). “Do Divorcing Couples Become Happier by Breaking Up?”. Journal of the Royal Statistical Society,169, 319-336.
  • En ligneGRAHAM, C. and FELTON, A. (2006). “Inequality and happiness: Insights from Latin America”. Journal of Economic Inequality, 4, 107-122.
  • En ligneGOEDHART, T., HALBERSTADT, V., KAPTEYN, A. and VAN PRAAG, B.M. (1977). “The poverty line: Concept and measurement”. Journal of Human Resources, 12, 503-520.
  • GUILLEN-ROYO, M. (2010), “Reference group consumption and the subjective wellbeing of the poor in Peru”, Journal of Economic Psychology, à paraître.
  • GUVEN, C., SENIK, C., and STICHNOTH, H. (2010). “You can’t be happier than your wife. Happiness Gaps and Divorce”. IZA, Discussion Paper No. 4599.
  • HAGERTY, M.R., and VEENHOVEN, R. (2000). “Wealth and Happiness Revisited - Growing Wealth of nations does go with greater happiness”. University of Cali-fornia-Davis, mimeo.
  • En ligneHELLIWELL, J.F. (2003). “How’s Life? Combining Individual and National Variables to Explain Subjective Well-Being”. Economic Modelling,20, 331-360.
  • HERRERA, J., RAZAFINDRAKOTO, M. and ROUBAUD, F. (2006). “The determinants of subjective poverty: A comparative analysis in Madagascar and Peru”, DIAL Working Paper 2006-01.
  • HUNG-LIN T. and CHIU S. “Income growth or equal income distribution? Implications of the relative income effect on happiness”. Department of economics Soochow university of Taiwan Working Paper.
  • En ligneINGLEHART, R., FOA, R., PETERSON, C., and WELZEL, C. (2008). “Development, Freedom, and Rising Happiness: A Global Perspective (1981–2007)”. Perspectives on Psychological Science,3, 264-285.
  • INGLEHART, R., and RABIER, J.R. (1986). “Aspirations Adapt to Situations - But Why Are the Belgians so Much Happier than the French? A Cross-Cultural Analysis of the Subjective Quality of Life”. In F.M. Andrews (Ed.), Research on the Quality of Life. Ann Arbor: Institute for Social Research University of Michigan.
  • INTER-AMERICAN DEVELOPMENT BANK (2008). Beyond Facts: Understanding Quality of Life. Cambridge, Mass.: Harvard University Press.
  • En ligneJOHANNSSON-STENMAN, O., CARLSSON, F., and DARUVALA, D. (2002). “Measuring Future Grandparents’ Preferences for Equality and Relative Standing”. Economic Journal,112, 362-383.
  • En ligneKAHNEMAN D. and DEATON A., (2010). “High income improves evaluation of life but not emotional well-being”, PNAS, Princeton Center for Health and Well-Being.
  • En ligneKAHNEMAN, D., and TVERSKY, A. (1979). “Prospect Theory: An Analysis of Decision Under Risk”. Econometrica,47, 263-291.
  • En ligneKENNY C. (2005). “Does development make you happy? Subjective well-being and economic growth in developing countries”. Social Indicators Research, 73, 199-219.
  • En ligneKINGDON, G., and KNIGHT, J. (2007). “Community, Comparisons and Subjective Well-being in a Divided Society”. Journal of Economic Behavior and Organization,64, 69-90.
  • KNIGHT J. and GUNATILAKA R., (2009). “Income, Aspirations and the Hedonic Treadmill in a Poor Society,” University of Oxford Discussion Paper N°468.
  • KNIGHT, J., and GUNATILAKA, R. (2010a). “Great Expectations? The Subjective Well-being of Rural–Urban Migrants in China”. World Development,38, 113-124.
  • KNIGHT, J., and GUNATILAKA, R. (2010b). “The rural-urban divide in China: Income but not happiness?” Journal of Development Studies, 46, 506-29.
  • En ligneKNIGHT J., SONG L. and GUNATILAKA R. (2007). “Subjective well-being and its determinants in rural China”. China Economic Review,20, 635-649.
  • KRISTENSEN, N. and WESTERGAARD-NIELSEN, N. (2006). “Job satisfaction and quits - Which job characteristics matters most?” Danish Economic Journal, 144, 230-248.
  • En ligneKUEGLER, A. (2009). “A Curse of Comparison? Evidence on Reference Groups for Relative Income Concerns”. World Bank, Policy Research Working Paper Series No. 4820.
  • LAYARD R. (2005). “Happiness: Lessons from a New Science”. Penguin, London.
  • En ligneLAYARD R., MAYRAZ G. and NICKELL S. (2008). “The Marginal Utility of Income”, Journal of Public Economics, 92, 1846-1857.
  • LAYARD R., MAYRAZ G. and NICKELL S., (2010). “Does Relative Income Matter? Are the Critics Right?” in E. Diener, J. F. Helliwell, and D. Kahneman (ed.), International Differences in Well-Being, Oxford University Press.
  • LUTTMER, E. (2005). “Neighbors as Negatives: Relative Earnings and Well-Being”. Quarterly Journal of Economics,120, 963-1002.
  • En ligneMCBRIDE, M. (2001). “Relative-income Effects on Subjective Well-being in the Cross-section”. Journal of Economic Behavior and Organization,45, 251-278.
  • MISHRA, V., NIELSEN, I. and SMYTH, R. (2010). “Relative Income, Temporary Life Shocks and Subjective Wellbeing in the Long-run”, Monash University, mimeo.
  • En ligneOSWALD, A.J. (1997). “Happiness and economic performance”. Economic Journal,107, 1815-31.
  • En lignePALMORE, E. (1969). “Predicting Longevity: A Follow-Up Controlling for Age”. Gerontologist,9, 247-50.
  • PISCHKE, S. (2010). “Money and Happiness: Evidence from the Industry Wage Structure”. LSE, mimeo.
  • POLITY IV PROJECT: Political Regime Characteristics and Transitions, 1800-2009. Monty G. Marshall, Director Monty G. Marshall and Keith Jaggers, Principal Investigators, Center for Systemic Peace and Colorado State University; Ted Robert Gurr, Founder, University of Maryland, http://www.systemicpeace.org/polity/polity4.htm.
  • PRADHAN, M. and RAVALLION, M. (1998). “Measuring poverty using qualitative perceptions of welfare”. World Bank and the University of Amsterdam Working paper.
  • En lignePRITCHETT, L. and SUMMERS, L. (1996). “Wealthier is Healthier.” Journal of Human Resources, 31, 841-868.
  • En ligneRAVALLION M. and LOKSHIN M. (2010). “Who cares about relative deprivation?” Journal of Economic Behavior & Organization, 73, 171-185.
  • En ligneRAYO, L., and BECKER, G.S. (2007). “Evolutionary Efficiency and Happiness”. Journal of Political Economy,115, 302-37.
  • ROJAS, M. (2010). “Relative Income and Well-Being in Latin America”, Universidad Popular Autónoma del Estado de Puebla, mimeo.
  • ROJAS, M. and JIMÉNEZ, E. (2007). “Pobreza Subjetive en México: El Papel de las Normas de Evaluación de Ingreso”, Universidad Popular Autónoma del Estado de Puebla, mimeo.
  • En ligneSANDVIK, E., DIENER, E. and SEIDLITZ, L. (1993). Subjective well-being: the convergence and stability of self and non self report measures. Journal of Personality, vol. 61, pp. 317-342.
  • SEN, A. (2001). Development as Freedom. New York: Alfred Knopf.
  • En ligneSENIK, C. (2004). “When Information Dominates Comparison: A Panel Data Anal-ysis Using Russian Subjective Data”. Journal of Public Economics,88, 2099-2123.
  • En ligneSENIK, C. (2005). “Income Distribution and Well-Being: What Can we Learn from Subjective Data?”. Journal of Economic Surveys,19, 43-63.
  • En ligneSENIK, C. (2008). “Ambition and jealousy. Income interactions in the “Old” Europe versus the “New” Europe and the United States”. Economica,75, 495-513.
  • En ligneSENIK C. (2009). “Income Distribution and Subjective Happiness: A Survey”, OECD Social Employment and Migration Working Papers n° 96. Published as Income distribution and subjective well-being, in OECD, 2009, Subjective well-being and social policy.
  • SMYTH, R. and QIAN, X. (2008). “Inequality and Happiness in Urban China.” Economics Bulletin, 4, 1-10.
  • En ligneSOLNICK, S., and HEMENWAY, D. (2005). “Are Positional Concerns Stronger in Some Domains than in Others?”. American Economic Review,95, 147-151.
  • En ligneSTARK, O., and TAYLOR, J.E. (1991). “Migration Incentives, Migration Types: The Role of Relative Deprivation”. Economic Journal,101, 1163-1178.
  • En ligneSTEVENSON, B., and WOLFERS, J. (2008), “Economic Growth and Subjective Well-Being: Reassessing the Easterlin Paradox”, Brookings Papers on Economic Activity, Spring.
  • En ligneSTUTZER, A. (2004), “The Role of Income Aspirations in Individual Happiness” Journal of Economic Behaviour and Organization 54, 1, 89-109.
  • En ligneTAKAHASHI, H., KATO, M., MATSUURA, M., MOBBS, D., SUHARA, T., and OKUBO, Y. (2009). “When your gain is my pain and your pain is my gain: Neural correlates of envy and Schadenfreude”. Science,323, 937-939.
  • En ligneURRY, H.L., NITSCHKE, J. B., DOLSKI, I., JACKSON, D. C., DALTON, K. M., MUELLER, C. J., ROSENKRANZ, M.A., RYFF, C. D., SINGER, B.H. and DAVIDSON, R.J. (2004). Making a life worth living: Neural correlates of well-being. Psychological Science, vol. 15, pp. 367-372.
  • En ligneVAN DE STADT, H., KAPTEYN, A., and VAN DE GEER, S. (1985). “The Relativity of Utility: Evidence from Panel Data”. Review of Economics and Statistics,67, 179-187.
  • En ligneVAN PRAAG, B.M. (1971). “The Welfare Function of Income in Belgium: An Empirical Investigation”. European Economic Review,2, 337-369.
  • En ligneVAN PRAAG, B.M. (1991). “Ordinal and Cardinal Utility”. Journal of Econometrics,50, 69-89.
  • VAN PRAAG, B. and FRIJTERS, P. (1999), The measurement of welfare and well-being: the Leyden approach” in D. Kahneman, E. Diener and N. Schwarz Wellbeing; The Foundations of Hedonic Psychology, New York: Russell Sage Foundation.
  • VENDRIK, M., and WOLTJER, G. (2006). “Happiness and Loss Aversion: When Social Participation Dominates Comparison”. IZA, Discussion Paper No. 2218.
  • En ligneWOLFERS, J. (2003). “Is Business Cycle Volatility Costly? Evidence from Surveys of Subjective Well-being”. International Finance,6, 1-26.
  • WOLFHARD K., 2010. “Conspicuous Consumption and Race: Evidence from South Africa,” Papers on Economics and Evolution 2010-03, Max Planck Institute of Economics, Evolutionary Economics Group.
  • En ligneZIZZO, D.J., and OSWALD, A.J. (2001). “Are People Willing to Pay to Reduce Others’ Incomes?”. Annales d’Economie et de Statistique,63-64, 39-65.

Comment le PIB mesure la croissance économique ?

On mesure la croissance économique par le taux de croissance d'un agrégat qui est depuis quelques décennies le produit intérieur brut (PIB).

Pourquoi le PIB Est

Le PIB est une mesure de la production d'une économie. Cette mesure est basée sur un système d'informations performant qui donne une vision juste de l'état d'une économie.

Quel est le lien entre la croissance économique et le développement ?

La croissance obéit à deux facteurs d'offre: le travail et le capital. Elle croit si la productivité de ces facteurs augmentent. Et le développement est une transformations qualitatives des sociétés. C'est l'ensemble des changements économiques, sociaux qui résultent des transformations de la société industrielle.

Quels sont les facteurs qui influences la croissance du PIB ?

Les grands agrégats économiques associés au PIB sont le revenu national brut (RNB), la capacité ou le besoin de financement de la Nation, les grandes composantes de l'équilibre entre les éléments de l'offre (PIB, importations) et de la demande (consommation, investissement, exportations), la ventilation des facteurs de ...

Comment Peut

La croissance du PIB peut être décomposée en la somme des contributions de ses différentes composantes : dépenses de consommation des ménages, des institutions sans but lucratif au service des ménages et des administrations publiques, investissement, variations de stocks et solde commercial.

Qu'est

La croissance d'un pays se mesure à l'évolution de son Produit intérieur brut sur une période donnée : mois, trimestre, semestre ou année. On la calcule à « euro constant », c'est-à-dire en éliminant la hausse des prix. Le critère le plus significatif est la croissance du PIB par habitant.